Intro to Nursing - Exam 4 Concepts - Modules 13-16

अब Quizwiz के साथ अपने होमवर्क और परीक्षाओं को एस करें!

After a transurethral prostatectomy a patient returns to his room with a triple-lumen indwelling catheter and continuous bladder irrigation. The irrigation is normal saline at 150 mL/hr. The nurse empties the drainage bag for a total of 2520 mL after an 8-hour period. How much of the total is urine output? (Fill in the blank.) (Potter: Fundamentals of Nursing, 8th Edition, Chapter 45)

Answer: 1320 mL. The output is determined by calculating the amount of irrigation solution and subtracting that from the total output: 150 × 8 = 1200. Total output is 2520. 2520 - 1200 = 1320 urine output., 1320 mL, The output is determined by calculating the amount of irrigation solution and subtracting that from the total output: 150 × 8 = 1200. Total output is 2520. 2520 - 1200 = 1320 urine output."

The nurse is administering diphenoxylate with atropine (Lomotil) to a patient. Which should be included in the patient teaching regarding this medication? (Select all that apply.) a. Caution the patient to avoid laxative abuse. b. Record the frequency of bowel movements. c. Caution the patient against taking sedatives concurrently. d. Encourage the patient to increase fluids. e. Instruct the patient to avoid this drug if he or she has narrow-angle glaucoma. f. Teach the patient that the drug acts by drawing water into the intestine. (Kee: Pharmacology: A Patient-Centered Nursing Process Approach, 8th Edition, Chapter 47)

Answer: A, B, C, D, E.

The nurse is informing the patient that the health care provider has ordered a urine test for culture and sensitivity that will be obtained from the patient's indwelling Foley catheter. Which statement(s), if made by the patient, indicates that further instruction is needed? (Select all that apply.) A. "That's okay; you can just get a sample out of my urinary drainage bag." B. "I think my catheter comes apart from the tubing that goes to the collection bag. We can take it apart and hold a cup at the end of the catheter until you get enough urine for the test." C. "After you clamp my tubing, I'm probably going to need some pain medication." D. "It won't hurt me when you get the urine." E. "The doctor is checking to see if I have a UTI." F. "You will have to insert a new catheter to get a sterile specimen." (Potter: Evolve Course Module 16)

Answer: A, B, C, F. The urine specimen will be obtained from the sampling port of the urinary catheter system. The only time urine may be collected from the drainage bag is immediately after insertion. The urinary drainage system should remain closed to avoid the introduction of microorganisms that could ascend and cause a subsequent UTI. The patient may feel some pressure as the bladder fills with urine, but pain is unexpected. The syringe is attached to the needle free sampling port to obtain the urine sample rather than into the patient's body. Changing the catheter is unnecessary to obtain a sterile urine specimen. A sterile urine specimen for culture and sensitivity is used to determine the presence of infection and antibiotics that will be effective against the pathogenic organism.

When using ice massage for pain relief, which of the following are correct? (Select all that apply.) A. Apply ice using firm pressure over skin. B. Apply ice until numbness occurs and remove the ice for 5 to 10 minutes. C. Apply ice until numbness occurs and discontinue application. D. Apply ice for no longer than 10 minutes. (Potter: Fundamentals of Nursing, 8th Edition, Chapter 43)

Answer: A, B. Cold therapies are particularly effective for pain relief. Ice massage involves applying a frozen cup of ice firmly over the skin. When numbness occurs, remove the ice for usually 5 to 10 minutes.

A patient is admitted to the hospital with an acute gout attack. The nurse expects that which medication will be ordered to treat acute gout? a. colchicine b. allopurinol c. probenecid d. sulfinpyrazone (Kee: Pharmacology: A Patient-Centered Nursing Process Approach, 8th Edition, Chapter 25)

Answer: A.

A patient is having a migraine attack. The nurse should know that which drugs are used to treat migraine attacks? a. Triptans b. Anticonvulsants c. Tricyclic antidepressants d. Beta-adrenergic blockers (Kee: Pharmacology: A Patient-Centered Nursing Process Approach, 8th Edition, Chapter 26)

Answer: A.

A patient is postoperative day 3 after a total colectomy and ileostomy. He has a nasogastric tube to continuous suction and a Foley catheter to continuous drainage. The night nurse reports a high output from the ileostomy. The patient's pulse is irregular, and he reports leg weakness. Based on this situation, the nurse would correctly suspect what type of imbalance? a. Hypokalemia b. Hyperkalemia c. Hyponatremia d. Hypercalcemia (Kee: Pharmacology: A Patient-Centered Nursing Process Approach, 8th Edition, Chapter 16)

Answer: A.

A patient is taking aspirin for arthritis. Which adverse reaction should the nurse teach the patient to report to the health care provider? a. Tinnitus b. Seizures c. Sinusitis d. Palpitations (Kee: Pharmacology: A Patient-Centered Nursing Process Approach, 8th Edition, Chapter 25)

Answer: A.

A patient who has constipation is prescribed a bisacodyl (Dulcolax) suppository. Which explanation will the nurse use to explain the action of bisacodyl? a. Acts on smooth intestinal muscle to gently increase peristalsis b. Absorbs water into the intestines to increase bulk and peristalsis c. Lowers surface tension and increases water accumulation in the intestines d. Pulls salts into the colon and increases water in the feces to increase bulk (Kee: Pharmacology: A Patient-Centered Nursing Process Approach, 8th Edition, Chapter 47)

Answer: A.

Assessing a patient following IV morphine administration, the nurse notes cold, clammy skin; a pulse of 40 beats/min; respirations of 10 breaths/min; and constricted pupils. Which medication will the patient likely need next? a. naloxone (Narcan) b. meloxicam (Mobic) c. pentazocine (Talwin) d. propoxyphene (Darvon) (Kee: Pharmacology: A Patient-Centered Nursing Process Approach, 8th Edition, Chapter 26)

Answer: A.

A male patient returned from the operating room 6 hours ago with a cast on his right arm. He has not yet voided. Which action would be the most beneficial in assisting the patient to void? A. Suggest he stand at the bedside B. Stay with the patient C. Give him the urinal to use in bed D. Tell him that, if he doesn't urinate, he will be catheterized (Potter: Fundamentals of Nursing, 8th Edition, Chapter 45)

Answer: A. A man voids more easily in the standing position.

Your nursing manager distributes biweekly newsletters of ongoing unit or health care agency activities and posts minutes of committee meetings on a bulletin board in the staff break room. This is an example of: A. Staff communication. B. Problem-solving committees. C. Interdisciplinary collaboration. D. Nurse-physician collaborative practice. (Potter: Fundamentals of Nursing, 8th Edition, Chapter 21)

Answer: A. A manager's greatest challenge, especially if a work group is large, is communication with staff. Posting minutes in an accessible place for all staff ensures that all staff members receive the same message: the correct message. Minutes of committee meetings are usually in an accessible location for all staff to read.

The nurse collects the supplies for the dressing change for the patient in bed 1 and signs out the capillary blood glucose monitoring equipment to test the glucose of the patient in bed 2 before walking down the hall to the room. The nurse is displaying: A. Organizational skills. B. Use of resources. C. Priority setting. D. Clinical decision making. (Potter: Fundamentals of Nursing, 8th Edition, Chapter 21)

Answer: A. A strategy for improvement of organizational skills is to collect all needed equipment for nursing interventions and procedures before going to the patient's room. This prevents the nurse from having to make multiple trips to the supply room or nursing station. Being organized is about being effective and efficient.

When preparing the client who is undergoing urography with contrast, the nurse plans to administer which medication before the procedure? Assessment: flank pain, dysuria, bilateral knee pain. Diagnostics: BUN 54, Creatinine 2.4, Calcium 8.5. Medications: Captopril, Metformin, Acetylcysteine. A. Acetylcysteine B. Metformin C. Captopril D. Acetaminophen (Ignatavicius: Medical-Surgical Nursing, 7th Edition, Chapter 68)

Answer: A. Acetylcysteine (an antioxidant) may be used to prevent contrast-induced nephrotoxic effects; this client has kidney impairment demonstrated by increased creatinine.

An older adult is scheduled for a double-contrast barium enema. What is the priority health teaching the nurse will provide? A. "Be sure to take the laxative as prescribed after the test." B. "Drink a gallon of GoLYTELY the day before the test." C. "Do not take food or fluids for 24 hours before the test." D. "Tell the nurse if you have flatus after the test is completed." (Ignatavicius: Medical-Surgical Nursing, 7th Edition, Chapter 55)

Answer: A. After the test, the client should be instructed to take a laxative to clear the contrast from the body. Reference: p. 1187, Health Promotion and Maintenance

What is a common GI problem that older adult clients experience more frequently as they age? A. Decreased hydrochloric acid B. Excess lipase production C. Increased liver enzymes D. Increased peristalsis (Ignatavicius: Medical-Surgical Nursing, 7th Edition, Chapter 55)

Answer: A. Atrophy of the gastric mucosa causes a decreased ratio of gastrin-secreting cells to somatostatin-secreting cells. This results in a decrease in hydrochloric acid, causing decreased absorption of iron and vitamin B12.

Which of the following is a priority for a nurse to include in a teaching plan for a patient who desires self-management and alternative strategies? A. Body alignment and superficial heat and cooling. B. Patient-controlled analgesia (PCA) pump. C. Neurostimulation. D. Peripheral nerve blocks. (Giddens: Concepts for Nursing Practice, Concept 26)

Answer: A. Body alignment and thermal management are examples of nonpharmacologic measures to manage pain. They can be used individually or in combination with other nondrug therapies. Proper body alignment achieved through proper positioning can help prevent or relieve pain. Thermal measures such as the application of localized, superficial heat and cooling may relieve pain and provide comfort. PCA, neurostimulation, and peripheral nerve blocks are not totally self-managed or alternative therapies, because they are used under the direction of medical professionals.

The older adult woman who reports a change in bladder function says, "I feel like a child who sometimes pees her pants." What is the nurse's best response? A. "Have you tried using the toilet at least every couple of hours?" B. "How does that make you feel?" C. "We can fix that." D. "That happens when we get older." (Ignatavicius: Medical-Surgical Nursing, 7th Edition, Chapter 68)

Answer: A. By emptying the bladder on a regular basis, urinary incontinence from overflow may be avoided, which may give the client some sense of control.

The client who has fluid overload has been taking a diuretic for the past 2 days and now experiences these changes. Which change indicates to the nurse that the diuretic is effective? A. Weight loss of 7 pounds B. Heart rate increased from 72 to 80 C. Respiratory rate decreased from 20 to 16 D. Morning blood glucose decreased from 142 mg/dL to 110 mg/dL (Ignatavicius: Medical-Surgical Nursing, 7th Edition, Chapter 13)

Answer: A. Diuretic drugs cause water loss and are often prescribed for edema. One liter of water weighs 2.2 pounds. Weight loss is expected when the client gets rid of the body of excess water. Reference: p. 180, Safe and Effective Care Environment

Which of the following is the best example of documentation on a patient with a urinary catheter? A. Catheter care provided; no encrustation noted. Foley catheter patent and draining clear yellow urine to bedside drainage bag. B. Catheter care provided. 14 French catheter intact with approximately 30 mL urine in bedside drainage bag. C. Unable to palpate urinary bladder. Patent denies discomfort; indwelling catheter draining well. D. Patient instructed on signs and symptoms of UTI and how to prevent while catheterized. (Potter: Evolve Course Module 16)

Answer: A. Documentation should include the appearance of the urine whether clear or cloudy or with sediment and color.

A cancer client is receiving low-dose oral morphine but is reporting both "breakthrough" pain and constipation. What intervention does the nurse implement? A. Administers ordered docusate sodium (Colace) and gabapentin (Neurontin) B. Decreases the morphine (morphine sulfate) dosage for the client C. Gives the client a Fleet (sodium biphosphate) enema D. Records the client's bowel movements (Ignatavicius: Medical-Surgical Nursing, 7th Edition, Chapter 5)

Answer: A. Docusate is a stool softener, and gabapentin is an adjuvant for breakthrough pain.

While working in the outpatient procedure unit, an RN is assigned to these clients. Which client does the nurse assess first? A. 51-year-old who recently had an endoscopic retrograde cholangiopancreatography (ERCP) B. 54-year-old who is ready for discharge following a colonoscopy C. 58-year-old who has just arrived for basal gastric secretion and gastric acid stimulation testing D. 60-year-old with questions about an endoscopic ultrasound examination (Ignatavicius: Medical-Surgical Nursing, 7th Edition, Chapter 55)

Answer: A. ERCP requires conscious sedation, so the client needs immediate assessment of respiratory and cardiovascular status.

Before collecting a stool sample for occult blood, the nurse instructs the nursing assistive personnel to: A. Ask the patient to void. B. Wash the patient's perineum. C. Secure a sterile, specimen container. D. Plan to collect the first specimen of the day. (Potter: Fundamentals of Nursing, 8th Edition, Chapter 46)

Answer: A. Emptying the urinary bladder before collecting the stool sample prevents contamination of the specimen.

Which of the following assessments do you perform routinely when an older adult patient is receiving intravenous 0.9% NaCl? A. Auscultate dependent portions of lungs B. Check color of urine C. Assess muscle strength D. Check skin turgor over sternum or shin (Potter: Fundamentals of Nursing, 8th Edition, Chapter 41)

Answer: A. Excessive or too-rapid infusion of 0.9% NaCl (normal saline) causes extracellular fluid volume (ECV) excess with pulmonary vessel congestion and potential pulmonary edema, especially in older adults, who cannot adapt as rapidly to increased vascular volume. Overload of intravenous normal saline eventually increases urine volume if kidneys are functioning but may not change urine color. Assessment of muscle strength is appropriate for potassium imbalances, not ECV imbalances. Skin turgor is not a reliable assessment of ECV deficit in older adults.

A health care provider writes the following order for an opioidnaive patient who returned from the operating room following a total hip replacement. "Fentanyl patch 100 mcg, change every 3 days." Based on this order, the nurse takes the following action: A. Calls the health care provider, and questions the order B. Applies the patch the third postoperative day C. Applies the patch as soon as the patient reports pain D. Places the patch as close to the hip dressing as possible (Potter: Fundamentals of Nursing, 8th Edition, Chapter 43)

Answer: A. Fentanyl is 100 times more potent than morphine and not recommended for acute postoperative pain.

Following the initiation of a pain management plan, pain should be reassessed and documented on a regular basis as a way to evaluate the effectiveness of treatments. At a minimum, pain should be reassessed A. With each new report of pain. B. After administration of a stronger analgesic. C. Every 10 minutes. D. Every 60 minutes. (Giddens: Concepts for Nursing Practice, Concept 26)

Answer: A. Following the initiation of a pain management plan, pain should be reassessed and documented on a regular basis as a way to evaluate the effectiveness of treatments. At a minimum, pain should be reassessed with each new report of pain and before and after administration of analgesics.

Situation: A 70-year-old female is admitted to the hospital with heart failure, shortness-of-breath (SOB), and 3+ pitting edema in her lower extremities. Her medications are furosemide (Lasix), digoxin (Lanoxin), and an angiotensin-converting enzyme (ACE) inhibitor (Lotensin). She states that she stopped taking her Lasix because she did not think that it was helping her heart failure. Her physician orders furosemide (Lasix) 5 mg IV push. Ten (10) hours after receiving the Lasix, the client's potassium (K+) level is 2.5 mEq/L. Knowing all of the client's medications, what problem(s) does the nurse anticipate in this client? A. Clinical manifestations of digoxin toxicity B. Increased heart rate and blood pressure (BP) C. Increased signs of congestive heart failure (CHF) D. Signs and symptoms of hypernatremia (Ignatavicius: Medical-Surgical Nursing, 7th Edition, Chapter 13)

Answer: A. Hypokalemia increases the sensitivity of cardiac muscle to Lanoxin and may result in digoxin toxicity, even when the digoxin level is within the therapeutic range.

Situation: A 77-year-old woman is brought to the emergency department by her family after she has had diarrhea for 3 days. The family tells the nurse that she has not been eating or drinking well, but that she has been taking her diuretics for congestive heart failure (CHF). Laboratory results include a potassium level of 7.0 mEq/L. Which medication(s) does the nurse anticipate administering? A. Insulin (regular insulin) and dextrose (D20W) B. Loperamide (Imodium) C. Sodium polystyrene sulfonate (Kayexalate) D. Supplemental potassium (Ignatavicius: Medical-Surgical Nursing, 7th Edition, Chapter 13)

Answer: A. If potassium levels are high, a combination of 20 units of regular insulin in 100 mL of 20% dextrose may be prescribed to promote movement of potassium from the blood into the intracellular fluid (ICF).

The nurse is catheterizing a male patient and obtains a clear amber urine return. As the nurse begins to inflate the balloon the patient complains of pain and resistance is felt. What is the nurse's best action? A. Allow fluid to flow back into syringe, and advance the catheter a little more before attempting to reinflate. B. Have the patient take slow deep breaths inhaling through the nose and exhaling through the mouth. C. Lift penis to position perpendicular to patient's body, and apply light traction. D. Advance catheter to bifurcation of the drainage and balloon inflation port. (Potter: Evolve Course Module 16)

Answer: A. If resistance occurs when inflating balloon or the patient verbalizes or shows nonverbal signs of pain, the balloon may not be entirely within the bladder. Stop inflation; allow fluid to flow back into syringe, and advance the catheter a little more before reattempting to inflate. Having the patient take slow deep breaths may help the patient relax but do not help with resistance or pain. Lifting the penis to a perpendicular position and applying light traction is done to straighten the urethra. Advancement of catheter to bifurcation of the drainage and balloon inflation port ensures proper placement of catheter through the longer urethra of the male patient.

A patient had an indwelling catheter for 3 weeks. The patient had the catheter removed 3 hours ago and now complains of having to go to the bathroom frequently and that it is painful to void. Which instruction is appropriate for the nurse to give the patient? A. "This is a normal occurrence after having a catheter in place for more than several days." B. "It sounds like you have a UTI. I will notify your health care provider." C. "I will need to inspect your perineal area and wash and dry the area." D. "If these symptoms continue, I will notify your health care provider to see if we can reinsert the catheter." (Potter: Evolve Course Module 16)

Answer: A. If the catheter was in place for more than several days, the patient may experience dysuria (painful voiding) resulting from inflammation of the urethral canal. Because of decreased bladder muscle tone, the patient may urinate frequently. These symptoms should subside with time.

The nurse is inserting an indwelling Foley catheter in a male patient. The nurse asks the patient to bear down as if to void, and slowly inserts the catheter through the urethral meatus. The nurse advances the catheter and meets resistance. What is the nurse's best initial action at this time? A. Ask the patient to take slow deep breaths while inserting the catheter slowly. B. Withdraw the catheter and notify the health care provider. C. Apply more force to insert the catheter inward. D. Remove the catheter, apply more lubricant, and reinsert. (Potter: Evolve Course Module 16)

Answer: A. If there is resistance to catheter insertion, the nurse should have the patient take slow deep breaths to promote relaxation while inserting the catheter slowly. Another technique is to rest the nurse's arm against the patient's leg and ask him to relax. When the leg muscles begin to relax, the nurse may continue the insertion process. If there is persistent resistance to insertion, the patient may have an enlarged prostate. Then it is appropriate to notify the health care provider; a Coudé catheter, with a slightly curved end, may be needed to facilitate insertion.

Hospital leadership seeking Magnet™ status implemented a new governance structure, including a clinical nursing council that governs clinical nursing standards and care for the organization. This is an example of which leadership style? A. Shared leadership. B. Transformational leadership. C. Democratic leadership. D. Autocratic leadership. (Giddens: Concepts for Nursing Practice, Concept 37)

Answer: A. In shared leadership, employees are empowered to make decisions for the organization. In transformational leadership, the leader provides the vision. In democratic leadership, the leader involves the followers in decision making. The leader still makes the final decision. In autocratic leadership, the leader makes all the decisions.

Which urinary assessment information indicates the potential need for increased fluids in the client? A. Increased blood urea nitrogen B. Increased creatinine C. Pale-colored urine D. Decreased sodium (Ignatavicius: Medical-Surgical Nursing, 7th Edition, Chapter 68)

Answer: A. Increased blood urea nitrogen (BUN) can indicate dehydration.

Which of the following requires strict surgical asepsis? A. Insertion of a Foley catheter. B. Applying a condom catheter C. Performing catheter care D. Emptying a bedside drainage bag

Answer: A. Inserting a Foley catheter is an invasive procedure requiring aseptic technique.

A male patient with back and lower abdominal injuries from a motor vehicle accident is unable to void. His health care provider has ordered the insertion of a catheter to determine the amount of residual urine and then to remain in place to assist him with voiding during this post-trauma period. What type of urinary catheter should the nurse anticipate using? A. A Foley catheter B. A straight catheter C. A Coudé catheter D. A condom catheter (Potter: Evolve Course Module 16)

Answer: A. It is acceptable to use an indwelling catheter in this case to obtain the residual urine amount. This reduces the number of catheterizations to one, especially since the health care provider has ordered the catheter to remain in place to assist the patient with voiding until the pain subsides.

The nursing assistive personnel (NAP) reports leakage around a patient's urinary catheter. What action should the nurse take first? A. Attempt to reinflate the balloon. B. Increase the patient's fluid intake and reassess in 1 hour. C. Remove the catheter and replace with a smaller size. D. Obtain a urine specimen. (Potter: Evolve Course Module 16)

Answer: A. Leakage around the urinary catheter could indicate that the catheter inserted was too small or that the balloon failed. The nurse should attempt to reinflate the balloon and if this does not work, then replace the catheter. Increasing the patient's fluid intake would not improve a leaking catheter. If the patient had a low hourly urine output, this may be appropriate. If the cause is not a faulty balloon, the nurse may need to contact the health care provider to get an order for a larger catheter. The patient is not exhibiting symptoms of infection so there is no need for an order for any laboratory testing of a urine sample.

A nurse and a student nurse are talking about health care coordination. Which of the following statements made by the nurse would be correct? A. "Patients have needs beyond the health care system." B. "Patients should coordinate their own care." C. "Physicians are the only ones who coordinate care." D. "Community health nurses coordinate care for the patient." (Giddens: Concepts for Nursing Practice, Concept 47)

Answer: A. Many patients do have needs that go beyond what the health care system can offer, such as financial or social assistance. Patients do not have the health care knowledge to coordinate their care. Physicians and community health nurses are not the only ones involved in the coordination of health care.

Which client does the charge nurse assign to an experienced LPN/LVN working on the adult medical unit? A. 32-year-old who needs a nasogastric tube inserted for gastric acid analysis B. 36-year-old who needs teaching about an endoscopic retrograde cholangiopancreatography (ERCP) C. 40-year-old who will need administration of IV midazolam hydrochloride (Versed) during an upper endoscopy D. 46-year-old who was recently admitted with abdominal cramping and diarrhea of unknown causes (Ignatavicius: Medical-Surgical Nursing, 7th Edition, Chapter 55)

Answer: A. Nasogastric tube insertion is included in LPN/LVN education and is an appropriate task for an experienced LPN/LVN.

When assessing the older adult, the nurse teaches the older adult that which age-related change causes nocturia? A. Decreased ability to concentrate urine B. Decreased production of antidiuretic hormone C. Increased production of erythropoietin D. Increased secretion of aldosterone (Ignatavicius: Medical-Surgical Nursing, 7th Edition, Chapter 68)

Answer: A. Nocturia may result from decreased kidney-concentrating ability associated with aging.

A postoperative client reports, "I have pain from a mild headache." Which PRN medication does the nurse administer? A. Acetaminophen (Tylenol ) B. Hydromorphone (Dilaudid ) C. Midazolam (Versed) D. Oxycodone hydrochloride w/acetaminophen (Tylox) (Ignatavicius: Medical-Surgical Nursing, 7th Edition, Chapter 5)

Answer: A. Non-opioid analgesics such as Tylenol are the first line of therapy for mild to moderate pain.

A patient has newly diagnosed hyperparathyroidism. What should the nurse expect to find during an assessment at the beginning of the nursing shift? A. Lethargy and constipation from hypercalcemia. B. Positive Trousseau's sign from hypercalcemia. C. Lethargy and constipation from hypocalcemia. D. Positive Trousseau's sign from hypocalcemia. (Giddens: Concepts for Nursing Practice, Concept 7)

Answer: A. Parathyroid hormone (PTH) shifts calcium from the bones into the extracellular fluid (ECF). Excessive PTH causes hypercalcemia, which is manifested by lethargy and constipation. A positive Trousseau's sign is characteristic of hypocalcemia rather than hypercalcemia. Answers that indicate hypocalcemia are not correct, because PTH moves calcium into the ECF.

Which one of the following instructions is crucial for the nurse to give to both family members and the patient who is about to be started on a patient-controlled analgesia (PCA) of morphine? A. Only the patient should push the button. B. Do not use the PCA until the pain is severe. C. The PCA prevents overdoses from occurring. D. Notify the nurse when the button is pushed. (Potter: Fundamentals of Nursing, 8th Edition, Chapter 43)

Answer: A. Patient preparation and teaching are critical to the safe and effective use of PCA devices. Patients need to understand PCA and be physically able to locate and press the button to deliver the dose. Be sure to instruct family members not to "push the button" for the patient.

A postoperative client is receiving epidural analgesia and reports itching. What does the nurse do next? A. Gives a small amount of naloxone (Narcan) B. Gives diphenhydramine (Benadryl) C. Gives an antiemetic D. Calls the surgeon (Ignatavicius: Medical-Surgical Nursing, 7th Edition, Chapter 5)

Answer: A. Pruritus (itching) is a common side effect of epidural opioids and is first treated with a small amount of naloxone (Narcan).

A patient is admitted for lower gastrointestinal (GI) bleeding. What color of stool does the nurse anticipate the patient to have? A. Red B. Black C. Green D. Orange (Potter: Fundamentals of Nursing, 8th Edition, Chapter 46)

Answer: A. Red-colored stool indicates lower GI bleeding.

A postoperative patient is currently asleep. Therefore the nurse knows that: A. The sedative administered may have helped him sleep, but assessment of pain is still needed. B. The intravenous (IV) pain medication is effectively relieving his pain. C. Pain assessment is not necessary. D. The patient can be switched. (Potter: Fundamentals of Nursing, 8th Edition, Chapter 43)

Answer: A. Sedatives, antianxiety agents, and muscle relaxants have no analgesic effect; however, they can cause drowsiness and impaired coordination, judgment, and mental alertness and contribute to respiratory depression. It is important to avoid attributing these adverse effects solely to the opioid. You need to conduct a thorough reassessment.

A client who is using patient-controlled analgesia (PCA) is asleep. The nurse observes a family member pushing the PCA button for the sleeping client. What does the nurse say to the visitor? A. "Please allow the client to push the button when needed. B. "Please don't touch any equipment in the client's room." C. "Thank you. I am sure the client appreciated that." D. "The client is asleep and is not in pain." (Ignatavicius: Medical-Surgical Nursing, 7th Edition, Chapter 5)

Answer: A. The "PC" in "PCA" means "patient-controlled," so having someone else push the button and administer analgesia defeats the purpose. More important, this action could cause oversedation and possible serious safety issues.

While caring for a patient with cancer pain, the nurse knows that the World Health Organization (WHO) analgesic ladder recommends: A. Transitioning use of adjuvants with nonsteroidal antiinfl ammatory drugs (NSAIDs) to opioids. B. Using acetaminophen for refractory pain. C. Limiting the use of opioids because of the likelihood of side effects. D. Avoiding total sedation, regardless of how severe the pain is. (Potter: Fundamentals of Nursing, 8th Edition, Chapter 43)

Answer: A. The WHO analgesic ladder transitions from the use of nonopioids (NSAIDS) with or without adjuvants to opioids with or without adjuvants. Acetaminophen is recommended for lesser levels of pain. Side effects related to the use of opioids may be unavoidable but are treatable. Treatment for severe pain may result in some level of sedation.

A nurse is assessing a client who had abdominal surgery yesterday. What method provides the most accurate data about resumption of peristalsis in the client? A. Asking the client whether he or she has passed flatus (gas) B. Auscultating bowel sounds in all abdominal quadrants C. Counting the number of bowel sounds in each abdominal quadrant D. Observing the abdomen for symmetry and distention (Ignatavicius: Medical-Surgical Nursing, 7th Edition, Chapter 55)

Answer: A. The best and most reliable method for assessing the return of peristalsis following abdominal surgery is the client's report of passing flatus within the past 8 hours or stool within the past 12 hours.

A client is scheduled for a colonoscopy. What does the nurse tell the client to do before the procedure is performed? A. "Begin a clear liquid diet 12 to 24 hours before the test." B. "Do not eat or drink anything for 12 hours before the test." C. "Give yourself tap water enemas until the fluid returns are clear." D. "You will have to drink a contrast liquid 2 hours before the test." (Ignatavicius: Medical-Surgical Nursing, 7th Edition, Chapter 55)

Answer: A. The client is instructed to be on a liquid diet for 12 to 24 hours before a colonoscopy.

An RN is assessing a 70-year-old client admitted to the unit with severe dehydration. Which finding requires immediate intervention by the nurse? A. Client behavior that changes from anxious and restless to lethargic and confused B. Deep furrows on the surface of the tongue C. Poor skin turgor with tenting remaining for 2 minutes after the skin is pinched D. Urine output of 950 mL for the past 24 hours (Ignatavicius: Medical-Surgical Nursing, 7th Edition, Chapter 13)

Answer: A. The client's change in level of consciousness suggests poor cerebral blood flow or shrinkage or swelling of brain cells caused by fluid shifts within the brain cells. These changes indicate a need for immediate intervention to prevent further damage to cerebral function.

Which of the following could be considered negligence? A. A condom catheter is removed every 3 days. B. Clean gloves are worn to apply a condom catheter. C. Allowing a family caregiver to apply the condom catheter. D. Avoiding the use of barrier creams on the penile shaft. (Potter: Evolve Course Module 16)

Answer: A. The condom catheter should be removed and perineal care performed at least every 24 hours to prevent skin breakdown and/or infection. Applying a condom catheter does not require sterile technique. Clean gloves should be worn to prevent the transmission of microorganisms. The use of barrier creams should be avoided because they prevent the sheath from adhering to the penile shaft. Skin protectant may be used if prescribed.

The client has returned form a captopril renal scan. Which teaching should the nurse provide when the client returns? A. "Arise slowly and call for assistance when ambulating." B. "I must measure your intake and output (I&O)." C. "We must save your urine because it is radioactive." D. "I must attach you to this cardiac monitor." (Ignatavicius: Medical-Surgical Nursing, 7th Edition, Chapter 68)

Answer: A. The drug can cause severe hypotension during and after the procedure. Warn him or her to avoid rapid position changes and of the risk for falling as a result of orthostatic (positional) hypotension.

A postoperative client is vomiting and states, "I am having a lot of pain—about a 7 on a scale of 1 to 10." Which route of administration does the nurse choose to administer an analgesic to the client? A. Intravenous B. Oral C. Rectal D. Transdermal (Ignatavicius: Medical-Surgical Nursing, 7th Edition, Chapter 5)

Answer: A. The intravenous route is the best choice for fast relief of nausea and pain.

During change-of-shift report the nurse states that a patient has early renal failure and to be alert to this when administering medications. Why would this be a concern? A. The kidneys assist in the detoxification of medication metabolites. B. The patient may not be able to absorb all of the medications. C. The bladder acts as a filter to remove wastes and form urine. D. The kidneys are the primary site for medication metabolism. (Potter: Evolve Course Module 16)

Answer: A. The kidneys detoxify and eliminate by products of medication metabolism. If the kidneys are unable to perform this function, medication toxicity can develop. The nephron, the functional unit of the kidney, forms the urine. The bladder holds the urine until it is excreted. The liver is a primary site for medication metabolism.

A patient who is comatose is admitted to the hospital with an unknown history. Respirations are deep and rapid. Arterial blood gas levels on admission are pH, 7.20; PaCO2, 21 mm Hg; PaO2, 92 mm Hg; and HCO3-, 8. You interpret these laboratory values to indicate: A. Metabolic acidosis B. Metabolic alkalosis C. Respiratory acidosis D. Respiratory alkalosis (Potter: Fundamentals of Nursing, 8th Edition, Chapter 41)

Answer: A. The low pH indicates acidosis. The low PaCO2 is caused by the hyperventilation, either from primary respiratory alkalosis (not compatible with the measured pH) or as a compensation for metabolic acidosis. The low HCO3- indicates metabolic acidosis or compensation for respiratory alkalosis (again, not compatible with the measured pH). Thus metabolic acidosis is the correct interpretation.

Situation: A 68-year-old man is admitted to the hospital with dehydration. He has a history of atrial fibrillation, congestive heart failure (CHF), and hypertension. His current medications are digoxin (Lanoxin), chlorothiazide (Diuril), and potassium supplements. He tells a nurse that he has had flu-like symptoms for the past week and has been unable to drink for the past 48 hours. The nurse starts the client's IV and receives laboratory results, which include a potassium level of 2.7 mEq/L. The physician orders an IV potassium supplement. How does the nurse administer this medication? A. Added to an IV, not to exceed 20 mEq/hr B. Added to an IV, not to exceed 30 mEq/hr C. Rapid IV push, a 25-mEq dose D. Slow IV push, a 30-mEq dose (Ignatavicius: Medical-Surgical Nursing, 7th Edition, Chapter 13)

Answer: A. The maximum recommended infusion rate of potassium is 5 to 10 mEq/hr. This rate is never to exceed 20 mEq/hr under any circumstances.

The nurse takes the blood pressure of a client who is being treated for hypocalcemia. When the cuff is tightest, the client's hand develops palmar flexion. After the cuff is deflated, the hand remains in palmar flexion. What is the nurse's best first action? A. Apply oxygen. B. Alert the Rapid Response Team. C. Slow the calcium-containing IV solution. D. Turn the client onto the side opposite the hand with the flexion. (Ignatavicius: Medical-Surgical Nursing, 7th Edition, Chapter 13)

Answer: A. The palmar flexion response (Trousseau's sign) indicates a worsening of the hypocalcemia, and the client is becoming hyperreflexive. This condition is approaching the danger of tetany. Although the Rapid Response Team should be called, the best first action is to apply oxygen because the skeletal muscle response to hypocalcemia is more severe under hypoxic conditions. Slowing the calcium-containing IV solution would be the worst intervention. Positioning the client differently will not help resolve the hypocalcemia or the potential for tetany. Reference: p. 190, Safe and Effective Care Environment

Since removal of the patient's Foley catheter, the patient has voided 50 to 100 mL every 2 to 3 hours. Which action should the nurse take first? A. Check for bladder distention B. Encourage fluid intake C. Obtain an order to recatheterize the patient D. Document the amount of each voiding for 24 hours (Potter: Fundamentals of Nursing, 8th Edition, Chapter 45)

Answer: A. The patient may experience urinary retention after catheter removal. If amounts voided are small, checking for bladder distention is necessary.

Which assessments are most important for the nurse to perform when monitoring a client after a retrograde cystogram? A. Temperature and urine character B. Kidney tenderness and flank pain C. 24-hour urine volume, BUN and creatinine levels D. Angioedema and other indicators of systemic allergic response (Ignatavicius: Medical-Surgical Nursing, 7th Edition, Chapter 68)

Answer: A. The retrograde cystogram involves instilling a contrast dye directly into the bladder through the urethra. Infection is the primary concern from instruments entering the bladder. Reference: p. 1486, Safe and Effective Care Environment

When obtaining a sterile urinary catheter specimen, the sterile specimen container should be opened and the lid: A. Placed with the inside up B. Placed with the inside down C. Held, because setting it on a table without a draped surface will cause contamination D. Loosened but left on the container (Potter: Evolve Course Module 16)

Answer: A. The sterile inside of the cap should be facing up to avoid contamination. It would be difficult to manipulate and obtain the midstream urine specimen if the lid were still on.

A client is admitted to the hospital with elevated serum amylase and lipase levels and a decreased calcium level. Which GI health problem is indicated by these laboratory findings? A. Acute pancreatitis B. Cirrhosis C. Crohn's disease D. Diarrhea (Ignatavicius: Medical-Surgical Nursing, 7th Edition, Chapter 55)

Answer: A. These laboratory values are commonly found in clients with acute pancreatitis.

A female patient reports that she is experiencing burning on urination, frequency, and urgency. The nurse notes that a clean-voided urine specimen is markedly cloudy. The probable cause of these symptoms and findings is: A. Cystitis. B. Hematuria. C. Pyelonephritis. D. Dysuria.

Answer: A. Urine is cloudy in cystitis because of bacterial and white cells.

A 68-year-old female patient is admitted for knee replacement surgery with an expected hospital stay of 2 weeks. She has no known allergies. The health care provider has ordered an indwelling Foley catheter to be inserted preoperatively. Which catheter should the nurse choose? A. 14 French, 5-mL balloon, latex catheter B. Coude catheter C. 16 French plastic catheter D. 18 French, 5-mL balloon, latex catheter E. 8 French, 3-mL balloon, latex catheter F. 16 French, 30-mL balloon, silicon catheter (Potter: Evolve Course Module 16)

Answer: A. Women require a 14 to 16 French catheter; it is usually best to begin with the smaller size. A 5-mL balloon is a common size balloon. Latex and rubber catheters are recommended for use up to 3 weeks. A Coude (elbowed/curved) catheter is often used for males with prostatic hypertrophy. Plastic catheters are suitable only for intermittent use because of their inflexibility. Men require a 16 to 18 French catheter; this would be too large for Mrs. Kelly. Pure silicon or Teflon catheters are best suited for long-term use (2 to 3 months). The 8 French, 3-mL balloon, latex catheter is a pediatric catheter.

Identify the indicators of a UTI: (Select all that apply.) A. Cool and clammy skin B. Fever C. Urinary drainage D. Complaints of pain E. Hypothermia F. A feeling of bladder fullness G. Abdominal pressure and discomfort H. Cloudiness of the urine (Potter: Evolve Course Module 16)

Answer: B, D, G, H. Fever is an indication of infection. Complaints of pain or burning are indicative of a UTI as urine passes over inflamed tissues. The patient may feel abdominal pressure and discomfort with a UTI. Urine should be clear; cloudy urine may indicate the presence of bacteria or white blood cells in the urine.

Which of the following steps should you take before removing fluid from the balloon in a Foley catheter? (Select all that apply.) A. Attach a 2-mL syringe to the balloon port and aspirate the fluid. B. Attach a 10 mL or larger syringe to the balloon port and allow the water to passively fill the syringe. C. Attach a 10- or 20-mL syringe to the balloon port and forcibly aspirate the water. D. Cut the balloon port and allow the water to slowly drain into a sterile basin. E. Gently aspirate the syringe plunger if water remains in the balloon. (Potter: Evolve Course Module 16)

Answer: B, D. A 10 mL or larger syringe should be attached to the balloon port, and the water should be allowed to passively fill the syringe. Gentle aspiration, if necessary, is appropriate.

A patient complains of constipation and requires a laxative. In providing teaching for this patient, the nurse reviews the common causes of constipation, including which cause? a. Motion sickness b. Poor dietary habits c. Food intolerance d. Bacteria (Escherichia coli) (Kee: Pharmacology: A Patient-Centered Nursing Process Approach, 8th Edition, Chapter 47)

Answer: B.

A patient gained 10 lb in 2 days. It is determined that the weight gain is caused by fluid retention. The nurse correctly estimates that the weight gain may be equivalent to approximately how much fluid? a. 2 L b. 4 L c. 10 L d. 11 L (Kee: Pharmacology: A Patient-Centered Nursing Process Approach, 8th Edition, Chapter 16)

Answer: B.

A patient is admitted to the emergency department in respiratory depression following self-injection with hydromorphone. The admitting nurse knows that which drug will reverse respiratory depression caused by opioid overdose? a. fentanyl b. naloxone c. butorphanol d. sufenta (Kee: Pharmacology: A Patient-Centered Nursing Process Approach, 8th Edition, Chapter 26)

Answer: B.

A patient is receiving IV potassium supplements. What is the most important nursing implication when administering this drug? a. IV potassium supplements may only be administered via a central vascular access device. b. IV potassium must be diluted before administration. c. IV potassium must be chilled before administration. d. IV potassium preparations do not contain preservatives. (Kee: Pharmacology: A Patient-Centered Nursing Process Approach, 8th Edition, Chapter 16)

Answer: B.

A patient with nausea is taking ondansetron (Zofran). She asks the nurse how this drug works. The nurse is aware that this medication has which action? a. Enhances histamine1 receptor sites b. Blocks serotonin receptors in the CTZ c. Blocks dopamine receptors in the CTZ d. Stimulates anticholinergic receptor sites (Kee: Pharmacology: A Patient-Centered Nursing Process Approach, 8th Edition, Chapter 47)

Answer: B.

Which of the following statements is false? A. Leaders are made, not born. B. Leadership in health care is an act reserved only for designated administrators and managers. C. Clinical leaders are needed in every setting to ensure that quality patient care is delivered and that new evidence and research findings are adopted to improve patient care. D. It is important for nurses who aspire to be leaders. (Giddens: Concepts for Nursing Practice, Concept 37)

Answer: B. "Leaders are made, not born" is true because leaders must continuously develop leadership skills. Leadership skills in every setting will improve the quality of patient care. Nursing leaders are vitally important to improve patient care at the bedside.

The health care provider's order is 1000 mL 0.9% NaCl IV over 6 hours. Which rate do you program into the infusion pump? A. 125 mL/hr B. 167 mL/hr C. 200 mL/hr D. 1000 mL/hr (Potter: Fundamentals of Nursing, 8th Edition, Chapter 41)

Answer: B. 1000 mL divided by 6 hours is 166.7 mL/hr, which rounds to 167 mL/hr (if infusion pump accepts decimals, program it to 166.7 mL/hr).

Which client does the charge nurse on the adult medical unit assign to an RN who has floated from the outpatient GI unit? A. 38-year-old who needs discharge instructions after having an endoscopic retrograde cholangiopancreatography (ERCP) B. 40-year-old who needs laxatives administered and effectiveness monitored before a colonoscopy C. 43-year-old recently admitted with nausea, abdominal pain, and abdominal distention D. 50-year-old with epigastric pain who needs conscious sedation during a scheduled endoscopy procedure (Ignatavicius: Medical-Surgical Nursing, 7th Edition, Chapter 55)

Answer: B. A nurse who has experience with chronic GI problems will have experience and training in instructing clients on colonoscopy preparation.

The patient rates his pain as a 6 on a scale of 0 to 10, with 0 being no pain and 10 being the worst pain. The patient's wife says that he can't be in that much pain since he has been sleeping for 30 minutes. Which is the most accurate resource for assessing the pain? A. The patient's wife is the best resource for determining the level of pain since she has been with him continually for the entire day. B. The patient's report of pain is the best method for assessing the pain. C. The patient's health care provider has the best knowledge of the level of pain that the patient that should be experiencing. D. The nurse is the most experienced at assessing pain. (Potter: Fundamentals of Nursing, 8th Edition, Chapter 43)

Answer: B. A patient's self-report of pain is the single most reliable indicator of the existence and intensity of pain.

A health care provider has ordered an indwelling catheter to be inserted to bedside drainage. Which of the following is NOT an expected indication for Foley catheterization? A. Preoperative status. B. To determine urinary retention. C. To obtain accurate urinary output in a critically ill patient. D. To allow a pressure ulcer on the coccyx to heal in a patient with urinary incontinence. (Potter: Evolve Course Module 16)

Answer: B. A straight catheter or bladder scan may be used to determine urinary retention and would not be an indication for an indwelling catheter. Indications for an indwelling catheter include preparing for surgery, to obtain accurate output measurements, and for incontinent patients in which a wound needs to heal.

The NAP documents "Peri-care given" next to "Urinary Catheter" on a patient with an indwelling urinary catheter. What is the best explanation of what the NAP did after application of clean gloves? The NAP: A. washed the perineal area with soap and water and applied a topical antimicrobial ointment at the urethral meatus around the catheter. B. stabilized the catheter and washed the catheter with soap and water from where the catheter enters the meatus down 4 inches toward the drainage tubing. C. inserted the hub of syringe into balloon port allowing the sterile water to return passively into the syringe and slid the catheter out into a waterproof pad. D. obtained a squirt bottle of warm water and had the patient squirt it over their perineum while sitting on the toilet. (Potter: Evolve Course Module 16)

Answer: B. After routine perineal care is given with soap and water, the catheter is cleansed. While stabilizing catheter with dominant hand and using a clean washcloth, soap, and water, the catheter is cleaned in a circular motion along its length for about 10 cm (4 inches). Cleansing starts where the catheter enters the meatus and down toward the drainage tubing. The application of topical antimicrobial products is not effective in reducing meatal bacterial flora and reducing risk for UTI. Do not include them as a part of routine catheter care.

Which of the following statements is true? A. Communication is the least important function of leadership. B. Task leaders will communicate the most effective ways to accomplish the work. C. Autocratic leaders value feedback from their followers. D. Leaders who have little communication with their staff are socioemotional leaders. (Giddens: Concepts for Nursing Practice, Concept 37)

Answer: B. All answers except "Task leaders will communicate the most effective ways to accomplish the work" are false because communication is the most important function of leadership. Autocratic leaders do not seek feedback from their followers, because they make all the decisions. Socioemotional leaders communicate frequently to gage followers' feelings and emotions.

The nurse asks the nursing assistant to hold the legs of a female patient during a Foley catheter insertion. This is an example of a nurse displaying: A. Organizational skills. B. Use of resources. C. Time management. D. Evaluation. (Potter: Fundamentals of Nursing, 8th Edition, Chapter 21)

Answer: B. Appropriate use of resources is an important aspect of clinical care coordination. Resources in this case include members of the health care team. In any setting the patient care is administered more smoothly when staff members work together. The staff needs to ask for assistance, especially when there is an opportunity to make a procedure or activity more comfortable and safer for the patient.

A leader exhibiting the attribute of change is best portrayed by which of the following descriptions? A. The leader tells the staff that they must comply with the new program to be implemented. B. The leader communicates the vision and goals of the organization and how the departmental goals will serve these goals. C. The leader develops goals for the department that will promote the department at the cost of other departments' goals. D. The annual plan for the department is not changed, so the department will not have additional work. (Giddens: Concepts for Nursing Practice, Concept 37)

Answer: B. Aspects of leadership that influence change are communication, understanding the organizational culture, and considering alternate paths to achieve a goal. In telling staff that they must comply with a new program, the leader does not value communication and alternate paths to achieve a goal. The leader who develops goals for the department that will promote the department at the cost of others' goals does not value organizational culture. The leader who does not change the annual plan for the department does not value change and does not strive to achieve the goals of the organization.

The nurse has been called to make a home visit to a patient with a history of a spinal cord injury and an indwelling Foley catheter. The patient appears diaphoretic and his face is flushed. The nurse takes the patient's vital signs with the following results: Temperature 98.4°F, pulse 54, respirations 20 and blood pressure 160/100. The patient's head of the bed is elevated. What action should the nurse take next? A. Notify the health care provider. B. Check for any kinks in catheter tubing. C. Have the patient take slow deep breaths. D. Lower the head of the bed. (Potter: Evolve Course Module 16)

Answer: B. Autonomic dysreflexia is a medical emergency caused by bladder (or bowel) distention after spinal cord injury. The first action should be to assess for bladder fullness and drain the patient's bladder (i.e., empty bladder by removing any blockage and/or kinks in the catheter tubing). The patient's head should be elevated. If this does not resolve the patient's symptoms, the nurse may consult with the health care provider.

The nurses on the unit developed a system for self-scheduling of work shifts. This is an example of: A. Responsibility. B. Autonomy. C. Accountability. D. Authority. (Potter: Fundamentals of Nursing, 8th Edition, Chapter 21)

Answer: B. Autonomy is freedom of choice and responsibility for the choices. One type of autonomy for nurses is work autonomy. In work autonomy the nurse makes independent decisions about the work of the unit such as scheduling or unit governance.

A nurse is teaching a group of unlicensed assistive personnel (UAP) about fluid intake principles for older adults. What does the nurse tell them? A. "Be careful not to overload them with too many oral fluids." B. "Offer fluids that they prefer frequently and on a regular schedule." C. "Restrict their fluids in the evening hours if they are incontinent." D. "Wake them every 2 hours during the night with a drink." (Ignatavicius: Medical-Surgical Nursing, 7th Edition, Chapter 13)

Answer: B. Because of the decreased thirst mechanism, older adults should be offered oral fluids every 2 hours. The likelihood of their accepting the fluid increases if the fluid is one they prefer.

The nurse manager of a medical-surgical unit is completing assignments for the day shift staff. The client with which electrolyte laboratory value is assigned to the LPN/LVN? A. Calcium level of 9.5 mg/dL B. Magnesium level of 4.1 mEq/L C. Potassium level of 6.0 mEq/L D. Sodium level of 120 mEq/L (Ignatavicius: Medical-Surgical Nursing, 7th Edition, Chapter 13)

Answer: B. Because this client's calcium level is within normal limits, it is appropriate to assign the client to an LPN/LVN.

An older adult client is receiving furosemide (Lasix) for treatment of peripheral edema. Which nursing assessment data identify that the client is at risk for falling? A. Dry oral mucous membranes. B. Orthostatic blood pressure changes C. Pulse rate of 72 beats/min and bounding D. Serum potassium level of 4.0 mEq/L (Ignatavicius: Medical-Surgical Nursing, 7th Edition, Chapter 13)

Answer: B. Blood pressure decreases when changing positions. The client may not have sufficient blood flow to the brain, causing sensations of light-headedness and dizziness. This problem increases the risk for falling, especially in older adults.

You teach patients to replace sweat, vomiting, or diarrhea fluid losses with which type of fluid? A. Tap water or bottled water B. Fluid that has sodium (salt) in it C. Fluid that has K+ and HCO3- in it D. Coffee or tea, whichever they prefer (Potter: Fundamentals of Nursing, 8th Edition, Chapter 41)

Answer: B. Body fluid losses remove sodium-containing fluid from the body and can cause extracellular fluid volume deficit unless both the sodium and the water are replaced.

A client with chronic pain feels no relief with high-dose opioids and says, "I just can't manage living right now." What intervention does the nurse anticipate the health care provider will order for this client? A. Adding acetaminophen (Tylenol) B. Adding sertraline (Zoloft) as adjuvant therapy C. Increasing the opioid dose to control the pain D. Replacing the opioid with sertraline (Zoloft) for depression (Ignatavicius: Medical-Surgical Nursing, 7th Edition, Chapter 5)

Answer: B. Both tricyclic and other antidepressants such as sertraline (Zoloft) help treat the depression that can accompany chronic pain. They also stimulate the activity of endogenous opiates (endorphins and enkephalins) by increasing levels of the neurotransmitter serotonin. Perhaps the greatest advantage of this group of drugs is their sedative effect.

A client being discharged after hip replacement says, "I am going to use hypnosis instead of medication to manage my pain. I believe in mind over body." How does the nurse respond? A. "I will cancel your medication order." B. "That sounds like a great plan; can you tell me more about it?" C. "That sounds like a wonderful idea; and I think it will definitely work!" D. "Your plan will not work; people with your type of pain need narcotics." (Ignatavicius: Medical-Surgical Nursing, 7th Edition, Chapter 5)

Answer: B. Complementary and alternative therapies should supplement, not replace, medication management. The nurse needs to obtain more data about the client's plan.

What should the nurse teach the client who is undergoing a study using contrast media? A. "You will need to have anesthesia or sedation." B. "A feeling of heat or warmth occurs when the contrast is injected." C. "Expect your urine to have a pink or red tinge after the procedure." D. "You will not be able to eat or drink for 4 to 6 hours after the procedure." (Ignatavicius: Medical-Surgical Nursing, 7th Edition, Chapter 68)

Answer: B. Contrast medium causes a sensation of flushing, heat, or warmth as it circulates through the bloodstream.

A new medical resident writes an order for OxyContin SR 10 mg PO q12 hours prn. Which part of the order does the nurse question? A. The drug B. The time interval C. The dose D. The route (Potter: Fundamentals of Nursing, 8th Edition, Chapter 43)

Answer: B. Controlled- or extended-release opioid formulations such as OxyContin are available for administration every 8 to 12 hours ATC. Health care providers should not order these long-acting formulations prn.

The nurse recognizes that which of these is the best indicator of kidney function? A. BUN B. Creatinine C. AST D. Alkaline phosphatase (Ignatavicius: Medical-Surgical Nursing, 7th Edition, Chapter 68)

Answer: B. Creatinine excretion, the end product of muscle metabolism, remains relatively steady and therefore is the best indicator of renal function.

As adults age, which common physiologic change is likely to alter their hydration status? A. Adrenal gland growth B. Decreased muscle mass C. Increased thirst mechanism D. Poor skin turgor (Ignatavicius: Medical-Surgical Nursing, 7th Edition, Chapter 13)

Answer: B. Decreased muscle mass causes decreased total body water, thus altering hydration status in the older adult.

The nurse is caring for a 78-year-old man with diarrhea. Of the following problems, which is the most important to consider? A. Malnutrition B. Dehydration C. Skin breakdown D. Incontinence (Potter: Fundamentals of Nursing, 8th Edition, Chapter 46)

Answer: B. Diarrhea interferes with absorption time of digestive juices. With frequent loose, watery stools, dehydration becomes a major problem in the older adult.

The patient talks with the nurse about bladder health. One of the most important recommendations the nurse can make for bladder health is A. Eat foods high in fiber. B. Drink 6 to 8 glasses of noncaffeinated fluids daily. C. Exercise in the morning and evening. D. Visit the urologist once yearly. (Giddens: Concepts for Nursing Practice, Concept 14)

Answer: B. Drinking 6 to 8 glasses of noncaffeinated fluids daily helps with bladder health because urine is not stagnating in the bladder. Exercising and eating foods high in fiber help with bowel elimination but do not have an effect on urination. Visiting the urologist is good if there is a problem, but this is not the most important recommendation from the nurse.

A client develops fluid overload while in the intensive care unit. Which nursing intervention does the nurse perform first? A. Draws blood for laboratory tests B. Elevates the head of the bed C. Places the extremities in a dependent position D. Puts the client in a side-lying position (Ignatavicius: Medical-Surgical Nursing, 7th Edition, Chapter 13)

Answer: B. Elevating the head of the bed will ease breathing for the client.

A client has a routine sigmoidoscopy. What common complication is the nurse looking for in a postprocedure assessment? A. Excessive diarrhea B. Heavy bleeding C. Nausea and vomiting D. Severe rectal pain (Ignatavicius: Medical-Surgical Nursing, 7th Edition, Chapter 55)

Answer: B. Excessive or heavy bleeding is a possible complication following a sigmoidoscopy. It must be reported immediately to the health care provider.

Which of the following actions associated with Foley catheterization could cause a potential problem? The bedside drainage bag is attached to the bed frame. Keeping the foreskin retracted after catheterization. Failing to test the balloon by injecting fluid from prefilled sterile water syringe into the balloon port prior to insertion. Cleansing the far labial fold, the near labial fold, and directly over the center of urethral meatus using a new swab with each area. (Potter: Evolve Course Module 16)

Answer: B. Failure to reduce the foreskin after catheterization can result in paraphimosis (constriction of the foreskin). The bedside drainage bag should be attached to the bed frame and not the bed rails to avoid accidentally raising the rails (and the collection bag) above the level of the bladder allowing reflux of urine. Testing the balloon by injecting fluid from the prefilled sterile water syringe into the balloon port is no longer a common practice. Testing the balloon may stretch the balloon and lead to damage causing increased trauma on insertion. Cleansing the far labial fold, the near labial fold, and directly over the center of urethral meatus using a new swab with each area is the correct procedure for cleaning the female patient.

Which of these patients do you expect will need teaching regarding dietary sodium restriction? A. An 88-year-old with a fractured femur scheduled for surgery B. A 65-year-old recently diagnosed with heart failure C. A 50-year-old recently diagnosed with asthma and diabetes D. A 20-year-old with vomiting and diarrhea from gastroenteritis (Potter: Fundamentals of Nursing, 8th Edition, Chapter 41)

Answer: B. Heart failure commonly causes extracellular fluid volume (ECV) excess because diminished cardiac output reduces kidney perfusion and activates the renin-angiotensin-aldosterone system, causing the kidneys to retain Na+ and water. Dietary sodium restriction is important with heart failure because Na+ holds water in the extracellular fluid, making the ECV excess worse.

Your patient who has diabetic ketoacidosis is breathing rapidly and deeply. Intravenous (IV) fluids and other treatments have just been started. What should you do about this patient's breathing? A. Notify her health care provider that she is hyperventilating B. Provide frequent oral care to keep her mucous membranes moist C. Ask her to breathe slower and help her to calm down and relax D. Assess her for pain and request an order for a sedative (Potter: Fundamentals of Nursing, 8th Edition, Chapter 41)

Answer: B. Hyperventilation is a compensatory mechanism for metabolic acidosis and should be allowed to continue. Rapid breathing can make oral mucous membranes dry and cracked.

The charge nurse is making client assignments for the day shift. Which client would be best to assign to an LPN/LVN? A. A client who has just returned from having a kidney artery angioplasty B. A client with polycystic kidney disease who is having a kidney ultrasound C. A client who is going for a cystoscopy and cystourethroscopy D. A client with glomerulonephritis who is having a kidney biopsy (Ignatavicius: Medical-Surgical Nursing, 7th Edition, Chapter 68)

Answer: B. Kidney ultrasounds are noninvasive procedures without complications; the LPN/LVN can provide this care.

The nurse notes that the patient's Foley catheter bag has been empty for 4 hours. The priority action would be to: A. Irrigate the Foley. B. Check for kinks in the tubing. C. Notify the health care provider. D. Assess the patient's intake. (Potter: Fundamentals of Nursing, 8th Edition, Chapter 45)

Answer: B. Kinks in tubing prevent flow of urine. To keep the drainage system patent, check for kinks or bends in the tubing.

Stephanie is a 70-year-old retired schoolteacher who is interested in nondrug, mind-body therapies, self-management, and alternative strategies to deal with joint discomfort from rheumatoid arthritis. Which of the following options should you suggest for her plan of care, considering her expressed wishes? A. Using a stationary exercise bicycle and free weights and attending a spinning class. B. Using mind-body therapies such as music therapy, distraction techniques, meditation, prayer, hypnosis, guided imagery, relaxation techniques, and pet therapy. C. Drinking chamomile tea and applying icy/hot gel. D. Receiving acupuncture and attending church services. (Giddens: Concepts for Nursing Practice, Concept 26)

Answer: B. Mind-body therapies are designed to enhance the mind's capacity to affect bodily functions and symptoms and include music therapy, distraction techniques, meditation, prayer, hypnosis, guided imagery, relaxation techniques, and pet therapy, among many others. Although getting exercise, drinking chamomile tea and applying gels, and receiving acupuncture and attending church services may be beneficial, they are not classified as mind-body therapies in combination as specified in these answer choices.

A 65-year-old woman has fallen while sweeping her driveway, sustaining a tissue injury. She describes her condition as an aching, throbbing back. This is characteristic of: A. Neuropathic pain. B. Nociceptive pain. C. Chronic pain. D. Mixed pain syndrome. (Giddens: Concepts for Nursing Practice, Concept 26)

Answer: B. Nociceptive pain refers to the normal functioning of physiologic systems that leads to the perception of noxious stimuli (tissue injury) as being painful. Patients describe this type of pain as dull or aching, and it is poorly localized. Neuropathic pain is described as shooting, tingling, burning, or numbness that is constant in the extremities, as in diabetic neuropathy. Chronic pain lasts longer than 30 days and is characterized by a disease affecting brain structure and function, such as chronic headaches or open wounds. Mixed pain syndromes are caused by different pathophysiologic mechanisms such as a combination of neuropathic and nociceptive pain; this occurs in syndromes such as sciatica, spinal cord injuries, and cervical or lumbar spinal stenosis.

Which of the following signs or symptoms in an opioid-naïve patient is of greatest concern to the nurse when assessing the patient 1 hour after administering an opioid? A. Oxygen saturation of 95% B. Difficulty arousing the patient C. Respiratory rate of 10 breaths/min D. Pain intensity rating of 5 on a scale of 0 to 10 (Potter: Fundamentals of Nursing, 8th Edition, Chapter 43)

Answer: B. Opioid-naive patients may develop a rare adverse effect of respiratory depression, and sedation always occurs before respiratory depression.

A patient is being discharged home on an around-the-clock (ATC) opioid for chronic back pain. Because of this order, the nurse anticipates an order for which class of medication? A. Stool softener B. Stimulant laxative C. H2 receptor blocker D. Proton pump inhibitor (Potter: Fundamentals of Nursing, 8th Edition, Chapter 43)

Answer: B. Patients usually become tolerant to the side effects of opioids, with the exception of constipation. Routinely administer stimulant laxatives, not simple stool softeners, to prevent and treat constipation.

The nurse is assessing the patient's condom catheter. Which of the following most likely indicates the condom catheter should be removed? A. Patient complains of the leg bag feeling "heavy" while in bed. B. Redness and/or excoriation of the penis C. Patient's urine appears clear amber with ammonia smell. D. Less than 30 mL/hr of urinary output. (Potter: Evolve Course Module 16)

Answer: B. Redness and excoriation of the penis are both signs of impaired skin integrity. If the patient complains that the leg bag feels heavy while in bed, he may need to be changed to a bedside drainage bag while lying down. Normal urine appears clear amber and has an ammonia smell. If the hourly urinary output appears low, the catheter should be checked for twisting or kinks first. This does not necessarily mean the catheter must be removed.

The nursing instructor is reviewing the renal system and urinary catheterization with her students. Which statement, if made by a nursing student, indicates that further instruction is needed? A. "The urinary tract is considered to be sterile." B. "The nurse may use clean technique to insert an indwelling catheter." C. "The urge to void is felt when the bladder contains 150 to 200 mL in an adult." D. "The minimum average hourly urine output is 30 mL." (Potter: Evolve Course Module 16)

Answer: B. Sterile technique is used whether inserting a straight or indwelling urinary catheter. Patients may use clean insertion technique in the home setting for intermittent catheterization. When the patient is in an acute care or long-term care setting, sterile insertion technique is required because of the high risk for nosocomial infections. The urinary tract is sterile. The desire to urinate can be sensed when the bladder contains a smaller amount of urine (150 to 200 mL in an adult and 50 to 100 mL in a child). Minimum average hourly output is 30 mL.

A nurse is educating a group of older adults about screening for colorectal cancer. Which statement by a group member indicates the need for further clarification about these guidelines? A. "A barium enema every 5 years is a screening option." B. "I will need to have a routine colonoscopy every 5 years." C. "My routine flexible sigmoidoscopy every 5 years is OK." D. "The 'virtual' colonoscopy every 5 years is acceptable." (Ignatavicius: Medical-Surgical Nursing, 7th Edition, Chapter 55)

Answer: B. The 2010 guidelines indicate that routine screening with colonoscopy is performed every 10 years, not every 5 years. Other options are performed at 5-year intervals.

A patient with a history of a stroke that left her confused and unable to communicate returns from interventional radiology following placement of a gastrostomy tube. The health care provider's order reads as follows: "Vicodin 1 tab, per tube, q4 hours, prn." Which action by the nurse is most appropriate? A. No action is required by the nurse because the order is appropriate. B. Request to have the ordered changed to ATC for the first 48 hours. C. Ask for a change of medication to meperidine (Demerol) 50 mg IVP, q3 hours, prn. D. Begin the Vicodin when the patient shows nonverbal symptoms of pain. (Potter: Fundamentals of Nursing, 8th Edition, Chapter 43)

Answer: B. The American Pain Society (2003) states that, if you anticipate pain for most of the day, you should consider ATC administration. Insertion of a gastrostomy tube is painful. This patient will most likely experience pain for at least the next 48 hours.

The RN is caring for a client who has just had a kidney biopsy. Which of these actions should the nurse perform first? A. Obtain BUN and creatinine. B. Position the client supine. C. Administer pain medications. D. Check urine for hematuria. (Ignatavicius: Medical-Surgical Nursing, 7th Edition, Chapter 68)

Answer: B. The client is positioned supine for several hours after a kidney biopsy to decrease the risk for hemorrhage.

A client is admitted to the hospital with severe right upper quadrant (RUQ) abdominal pain. Which assessment technique does the nurse use for this client? A. Assesses the abdomen in the following sequence: inspection, palpation, percussion, auscultation B. Examines the right upper quadrant (RUQ) of the abdomen last C. Has the client lie in a supine position with legs straight and arms at the sides D. Views the abdomen by looking directly down while standing over the client's abdominal area (Ignatavicius: Medical-Surgical Nursing, 7th Edition, Chapter 55)

Answer: B. The client reports pain in the RUQ, so the nurse would examine this area last in the examination sequence. This sequence prevents the client from tensing abdominal muscles because of the pain, which would make the examination difficult.

A client with newly diagnosed irritable bowel syndrome (IBS) reports having five to six loose stools daily. What is the common psychological client response to this GI health problem? A. Acceptance B. Embarrassment C. Euphoria D. Grief (Ignatavicius: Medical-Surgical Nursing, 7th Edition, Chapter 55)

Answer: B. The client who has a new onset of IBS with frequent stools most likely would be embarrassed.

The family of a client with chronic cancer pain says to the nurse, "Can you please reduce Dad's pain medication so that we can spend more quality time with him?" How does the nurse respond? A. "I will ask his oncologist about your question." B. "Let's ask your father about your request." C. "No, his pain relief is more important than your concerns." D. "Yes, this is a valuable way for all of you to make needed adjustments." (Ignatavicius: Medical-Surgical Nursing, 7th Edition, Chapter 5)

Answer: B. The client's desires about analgesia are the most important consideration in this scenario. He should be consulted initially about his family's request. This open-ended type of question acknowledges the family, while keeping the client as the major decision maker.

The nurse is caring for a confused patient who is wearing a vest restraint in bed. The nurse speaks with an unlicensed assistant about toileting the patient. The nurse knows the unlicensed assistant understands the toileting procedure when she makes this statement: A. The patient must remain in the restraints all day. B. The patient needs to be toileted to maintain a regular toileting schedule. C. The patient needs to be provided with adult briefs for incontinence. D. The patient will use the call bell when he or she feels the urge to void. (Giddens: Concepts for Nursing Practice, Concept 14)

Answer: B. The correct answer is toileting the patient so he or she can maintain a normal toileting schedule. Leaving the patient in restraints all day is against the standard of care. Providing the patient with briefs when he or she is not incontinent does not meet the patient's toileting needs. If the patient is confused, he or she will not be able to use the call bell.

The nurse is reviewing urinary catheter care with a newly hired nursing assistive personnel (NAP). Which statement made by the NAP indicates further instruction is needed? A. "Urinary catheter care is a clean procedure; sterile gloves are an unnecessary expense." B. "The bedside drainage bag should only be emptied when it is full." C. "During catheter care, you should relocate the tape that anchors the catheter and replace it as necessary." D. "Condom catheter care can be delegated to NAP and family members." (Potter: Evolve Course Module 16)

Answer: B. The fluid collection bag should be emptied when two-thirds full, or at least once every 8 hours. Clean technique is used to perform catheter care and sterile gloves are unnecessary. Moving the tape prevents the skin from becoming irritated. After reviewing the signs of infection, characteristics of normal urine, and the proper procedure, this task can be delegated to NAP and/or family members.

A patient has a tumor that secretes excessive antidiuretic hormone (ADH). He is confused and lethagic. His partner wants to know how a change in blood sodium can cause these symptoms. What should the nurse teach the patient's partner? A. Decreased sodium in the blood causes the blood volume to decrease so that not enough oxygen reaches the brain. B. Decreased sodium in the blood causes brain cells to swell so that they do not work as effectively. C. Increased sodium in the blood causes the blood volume to increase so that too much oxygen reaches the brain. D. Increased sodium in the blood causes brain cells to shrivel so that they do not work as effectively. (Giddens: Concepts for Nursing Practice, Concept 7)

Answer: B. The normal action of ADH is renal reabsorption of water, which dilutes the blood. Excessive ADH causes hyponatremia, which is manifested by a decreased level of consciousness because the osmotic shift of water into the brain cells impairs their function. Hyponatremia does not decrease the blood volume. Answers that include increased sodium in the blood are incorrect because ADH excess causes hyponatremia rather than hypernatremia.

A nursing assistant reports that a client receiving PCA morphine is very drowsy and has a respiratory rate of 10 breaths/min. What is the nurse's best action at this time? A. Continue to monitor the client for further changes. B. Assess the client with a focus on sedation and respirations. C. Stop the PCA morphine infusion immediately. D. Call the physician to report the client's changes. (Ignatavicius: Medical-Surgical Nursing, 7th Edition, Chapter 5)

Answer: B. The nurse will need to perform his or her own assessment of the client with a focus on the client's reportedly low respiratory rate and drowsiness. The nurse cannot assume the ancillary staff's assessment is correct. Furthermore, drowsiness is an anticipated result of morphine administration. The client is demonstrating untoward effects from the morphine. The nurse should not stop the infusion of morphine until an assessment is performed. Although the nurse's assessment may eventually lead to a consultation with the physician, a thorough assessment of the client must be done first. Reference: p. 56, Physiological Integrity

A patient injured in an earthquake today when a wall fell on his legs received 9 units of blood an hour ago because he was hemorrhaging. Which laboratory value should the nurse check first when the report returns? A. Serum sodium. B. Serum potassium. C. Serum total calcium. D. Serum magnesium. (Giddens: Concepts for Nursing Practice, Concept 7)

Answer: B. The patient has two major risk factors for hyperkalemia: massive sudden cell death from a crushing injury (potassium shift from cells into the extracellular fluid) and massive blood transfusion (rapid potassium intake). Although massive blood transfusion may cause calcium and magnesium ions to bind to citrate in the blood, thereby decreasing the physiologic availability of those ions, it does not decrease the total calcium or magnesium laboratory measurements. Clinically significant changes in serum sodium are the least likely in this patient.

The nurse has inserted a catheter 7.5 cm (3 inches) in a female patient and obtains no urine return even though her bladder is distended. What action should the nurse take at this time? A. Remove the catheter and have another nurse attempt to catheterize the patient. B. Leave the catheter in vagina as a landmark and insert another sterile catheter. C. Remove the catheter and reinsert into the urethra. The nurse may straighten the urethra by inserting one finger of sterile gloved hand inside the vagina and applying gentle pressure upward. D. Inflate the balloon and reassess in 1 hour for urine return in the bedside drainage bag. (Potter: Evolve Course Module 16)

Answer: B. There should be a urine return since the patient's bladder is distended. If no urine appears, catheter may be in vagina. If misplaced, the nurse should leave the catheter in vagina as a landmark where not to insert, and insert another sterile catheter. The nurse may straighten the urethra by inserting one finger of sterile gloved hand inside the vagina and applying gentle pressure upward; however, a new sterile catheter should be used.

When coordination and continuity of health care are transferred between different locations or different levels of care within the same location, this is identified as A. Health care doctrine. B. Transitional care. C. Transactional care. D. Multilevel care. (Giddens: Concepts for Nursing Practice, Concept 47)

Answer: B. Transitional care transfers between different locations, as described in the chapter. Health care doctrine, transactional care, and multilevel care do not fit the definition of coordination and continuity of care.

A 67-year-old client who had an abdominal x-ray as part of pre-admission testing for a gastrointestinal problem has just been told that he has a horseshoe-shaped kidney. He is very upset, telling the nurse that he has never had any health problems until the past month and now feels that he is "falling apart." What is the nurse's best response? A. Remind him that it was lucky that he was being x-rayed anyway and that the problem was found at an early stage. B. Reassure him that it is unlikely that the kidney shape is important since he has not had other kidney problems. C. Ask him whether anyone else in his family has ever been diagnosed with a horseshoe-shaped kidney. D. Reassure him that his health care provider will request a consultation with a kidney specialist. (Ignatavicius: Medical-Surgical Nursing, 7th Edition, Chapter 68)

Answer: B. Variations in the number and shape of the kidneys are relatively common. Most variations are not harmful and do not require further assessment. The fact that this client is 67 years old and has had few health problems is a good indication that this is just an incidental finding on x-ray and has no meaning for his kidney health. Reference: p. 1476, Psychosocial Integrity

As part of catheter insertion assessment, where should the nurse palpate? A. At the costovertebral angle B. Above the symphysis pubis C. Starting at the right iliac crest and moving upward along the midclavicular line D. Midway between the xyphoid process and symphysis pubis (Potter: Evolve Course Module 16)

Answer: B. When empty, the bladder is difficult to locate and palpate; if it is full, it may be palpated as a dome-shaped structure above the symphysis pubis. If the bladder is severely distended, it may extend into the abdomen. The costovertebral angle is formed by the last rib and vertebral column and is a landmark used during kidney palpation. Palpating the kidneys is unrelated to bladder fullness. The nurse may use percussion starting at the right iliac crest and move upward along the miclavicular line to determine the size of the liver. Liver size is unrelated to bladder fullness. The area of the umbilicus is located between the xiphoid process and sumphysis pubis.

Which percussion technique does the nurse use to assess the client with reports of flank pain? A. Places fingers outstretched over the flank area and percusses with fingertips B. Places one hand with palm down flat over the flank area and uses the other fisted hand to thump the hand on the flank C. Places one hand with the palm up over the flank area and cups the other hand to percuss the hand on the flank D. Quickly taps the flank area with cupped hands (Ignatavicius: Medical-Surgical Nursing, 7th Edition, Chapter 68)

Answer: B. While the client assumes a sitting, side-lying, or supine position, form one of the hands into a clenched fist. Place the other hand flat over the costovertebral (CVA) angle of the client. Then, quickly deliver a firm thump to the hand over the CVA area.

5. When metoclopramide (Reglan) is given for nausea, the nurse plans to caution the patient to avoid which substance? a. Milk b. Coffee c. Alcohol d. Carbonated beverages (Kee: Pharmacology: A Patient-Centered Nursing Process Approach, 8th Edition, Chapter 47)

Answer: C.

A patient is receiving 10 mEq of KCl in 100 mL of NS to infuse over 1 hour IVPB. The patient has a #22-gauge peripheral IV in his right forearm. The patient reports pain at the insertion site; the nurse notes that the site is reddened, warm, and tender to the touch. Which action would the nurse take? a. Aspirate and check for blood return, and then slow the IV rate. b. Discontinue the IV, and then have a central line inserted to administer the potassium. c. Stop the infusion, and discontinue the IV immediately. d. Apply warm compresses to the IV site. (Kee: Pharmacology: A Patient-Centered Nursing Process Approach, 8th Edition, Chapter 16)

Answer: C.

A patient is taking ibuprofen. The nurse understands that COX-1 and COX-2 inhibitors are different in that ibuprofen is more likely than celecoxib to cause which adverse effect? a. Fever b. Constipation c. Peptic ulcer disease d. Metallic taste when eating (Kee: Pharmacology: A Patient-Centered Nursing Process Approach, 8th Edition, Chapter 25)

Answer: C.

A patient requires a nonopioid medication. The nurse knows that which medication will cause the least gastrointestinal distress? a. aspirin b. ketorolac c. celecoxib d. ibuprofen (Kee: Pharmacology: A Patient-Centered Nursing Process Approach, 8th Edition, Chapter 26)

Answer: C.

A patient states during a medical history that he takes several acetaminophen tablets throughout the day. The nurse teaches the patient that the dosage should not exceed which amount? a. 1 g/day b. 2 g/day c. 4 g/day d. 6 g/day (Kee: Pharmacology: A Patient-Centered Nursing Process Approach, 8th Edition, Chapter 26)

Answer: C.

For the patient receiving periodic morphine IV push, which is the most critical finding for the nurse to discover? a. Increased temperature b. Decreased bowel sounds c. Decreased respirations d. Increased red blood cell count (Kee: Pharmacology: A Patient-Centered Nursing Process Approach, 8th Edition, Chapter 26)

Answer: C.

When teaching a patient who is receiving allopurinol, what should the nurse encourage the patient to do? a. Eat more meat. b. Increase vitamin C intake. c. Have annual eye examinations. d. Take medication 2 hours before meals. (Kee: Pharmacology: A Patient-Centered Nursing Process Approach, 8th Edition, Chapter 25)

Answer: C.

The nurse has received an order to insert a Foley catheter in a 24-year-old female patient. Which catheter would be most appropriate for this patient? A. 12 Fr 5 mL balloon B. 10 Fr 3 mL balloon C. 14 Fr 5 mL balloon D. 16 Fr 30 mL balloon E. 18 Fr 5 mL balloon (Potter: Evolve Course Module 16)

Answer: C. A 14 to 16 Fr catheter is indicated for adult women; the smaller size catheter should be chosen first to prevent urethral trauma. 8 to 10 Fr with 3-mL balloon is generally used with children. 16 to 18 Fr with 5-mL balloon is generally used with men.

The nurse understands that, when comparing nasogastric tubes used for gastric decompression, a Salem sump is specifically designed to: A. Minimize the risk of a bowel obstruction. B. Ensure drainage of the intestines. C. Prevent gastric mucosal damage. D. Promote resting the gut. (Potter: Fundamentals of Nursing, 8th Edition, Chapter 46)

Answer: C. A Salem sump tube has a double lumen. The second lumen is the blue pig-tailed portion that is open to air for the purpose of equalizing the pressure outside the body to inside the stomach. This prevents the tip of the Salem sump from becoming attached to the stomach lining, thus preventing mucosal irritation and bleeding.

Which instruction does the nurse give the client who needs a clean catch urine specimen? A. Save all urine for 24 hours. B. I will collect the first specimen of the morning. C. Do not touch the inside of the container. D. You will receive an isotope injection, then I will collect your urine. (Ignatavicius: Medical-Surgical Nursing, 7th Edition, Chapter 68)

Answer: C. A clean catch specimen is used to obtain urine for culture and sensitivity of organisms present; contamination by the client's hands will alter the specimen and results.

A client has a low serum potassium level and is ordered a dose of parenteral potassium chloride (KCl). How does a nurse safely administer KCl to the client? A. Administers 5 mEq, intramuscularly (IM) B. Dilutes 200 mEq in 1 liter of normal saline and infuses at 100 mL/hr C. Infuses 10 mEq over a 1-hour period D. Pushes 5 mEq through a central access line (Ignatavicius: Medical-Surgical Nursing, 7th Edition, Chapter 13)

Answer: C. A dose of KCl 10 mEq given over 1 hour is appropriate for this client.

A client with hyperkalemia is being treated with drugs to improve the condition. Which potassium level indicates that therapy is effective? A. 7.6 mEq/L B. 5.6 mEq/L C. 4.6 mEq/L D. 2.6 mEq/L (Ignatavicius: Medical-Surgical Nursing, 7th Edition, Chapter 13)

Answer: C. A potassium level of 4.6 mEq/L is a normal level, indicating that therapy was effective. Normal levels are 3.5 to 5.0 mEq/L.

A nurse is reviewing serum electrolytes and blood chemistry for a newly admitted client. Which result causes the greatest concern? A. Glucose: 97 mg/dL B. Magnesium: 2.1 mEq/L C. Potassium: 5.9 mEq/L D. Sodium: 143 mEq/L (Ignatavicius: Medical-Surgical Nursing, 7th Edition, Chapter 13)

Answer: C. A potassium value of 5.9 mEq/L is high, and the client should be assessed further.

A nurse is planning a dressing change on a postoperative mastectomy client. The client is receiving acetaminophen and oxycodone (Percocet) orally for pain every 4 hours and is due to receive them at 4 PM. When does the nurse change the dressing? A. 3:30 PM B. 4:00 PM C. 4:30 PM D. 7:00 PM (Ignatavicius: Medical-Surgical Nursing, 7th Edition, Chapter 5)

Answer: C. About 30 minutes after an analgesic is received is an optimal time to perform a procedure on a client. The opioid has had time to take effect and provide relief for the client.

While administering medications, the nurse realizes that she has given the wrong dose of medication to a patient. She acts by completing an incident report and notifying the patient's health care provider. The nurse is exercising: A. Authority. B. Responsibility. C. Accountability. D. Decision making. (Potter: Fundamentals of Nursing, 8th Edition, Chapter 21)

Answer: C. Accountability means that nurses are answerable for their actions. It means that they accept the commitment to provide excellent patient care and the responsibility for the outcomes of the actions in providing it. Following institutional policy for reporting errors demonstrates the nurse's commitment to safe patient care.

Your patient had 200 mL of ice chips and 900 mL intravenous (IV) fluid during your shift. Which total intake should you record? A. 700 mL B. 900 mL C. 1000 mL D. 1100 mL (Potter: Fundamentals of Nursing, 8th Edition, Chapter 41)

Answer: C. Add one half the volume of ice chips to other intake to calculate total intake.

The patient states that she "loses urine" every time she laughs or coughs. The nurse teaches the patient measures to regain urinary control. The nurse recognizes the need for further teaching when the patient states: A. "I will perform my Kegel exercises every day." B. "I joined weight watchers." C. "I drink two glasses of wine with dinner." D. "I have tried urinating every 3 hours." (Potter: Fundamentals of Nursing, 8th Edition, Chapter 45)

Answer: C. Alcohol is a bladder irritant. It increases urine production and causes uncontrolled bladder contractions.

The charge nurse on a medical-surgical unit is completing assignments for the day shift. Which client is assigned to the LPN/LVN? A. 44-year-old with congestive heart failure (CHF) who has gained 3 pounds since the previous day B. 58-year-old with chronic renal failure (CRF) who has a serum potassium level of 6 mEq/L C. 76-year-old with poor skin turgor who has a serum osmolarity of 300 mOsm/L D. 80-year-old with 3+ peripheral edema who has crackles throughout the posterior chest (Ignatavicius: Medical-Surgical Nursing, 7th Edition, Chapter 13)

Answer: C. Although the client has poor skin turgor, the serum osmolarity indicates that fluid balance is normal; this client is the most stable of the four clients described.

A postoperative client is requesting medication for pain every 4 hours. In planning effective pain management, what assessment question does the nurse ask the client before administering the medication? A. "Are you bleeding?" B. "Are you really hurting every 4 hours?" C. "Is your pain controlled between doses?" D. "What do you do for pain when you're at home?" (Ignatavicius: Medical-Surgical Nursing, 7th Edition, Chapter 5)

Answer: C. Asking the client about the frequency of pain and how the pain is being controlled helps in formulating an effective pain management plan.

The nurse is assisting the patient with coughing and deep-breathing exercises following abdominal surgery. This is which priority nursing need for this patient? A. Low priority B. High priority C. Intermediate priority D. Nonemergency priority (Potter: Fundamentals of Nursing, 8th Edition, Chapter 21)

Answer: C. Assisting the patient with cough and deep breathing is an intermediate priority. Intermediate priorities are nonemergency, nonlife-threatening actual or potential needs that the patient and family members are experiencing. Anticipating teaching needs of patients related to a new drug or taking measures to decrease postoperative complications are examples of intermediate priorities.

Which of the following actions, if made by the nurse, could be considered negligence? A. Leaving the drainage tubing below the catheter port clamped for 30 minutes. B. Using aseptic technique to obtain 5 mL of urine for a urine culture. C. Obtaining the urine specimen at 1030 and transporting it to the lab at 1115. D. Labeling the cup but not the lid and transporting the specimen in a biohazard bag. (Potter: Evolve Course Module 16)

Answer: C. Bacteria can multiply rapidly at room temperature. The specimen should be sent to the lab within 20 minutes or refrigerated for up to 2 hours. It is appropriate to clamp the drainage tubing below the catheter port for 30 minutes in order for urine to accumulate for collection. Aseptic technique should be used to prevent contamination of the specimen. At least 3 mL of urine is necessary to perform a urine culture. The cup should be labeled, not the lid to prevent errors related to incorrect identification.

Which client does the RN arriving for duty assess first? A. A 27-year-old who has chronic severe back pain with movement B. A 51-year-old with lung cancer who complains of pain "whenever I cough" C. A 56-year-old with acute pancreatitis who complains of increasing abdominal pain D. A 63-year-old who complains of ongoing pain associated with rheumatoid arthritis (Ignatavicius: Medical-Surgical Nursing, 7th Edition, Chapter 5)

Answer: C. Because acute pain is a biologic warning signal, the nurse should assess the client with pancreatitis for complications such as bleeding or perforation that may be causing the client's increasing pain.

A physician writes orders for a client who is admitted with a serum potassium (K) level of 6.9 mEq/L. What does the nurse implement first? A. Administering sodium polystyrene sulfonate (Kayexalate) orally. B. Ensuring that a potassium-restricted diet is ordered. C. Placing the client on a cardiac monitor. D. Teaching the client about foods that are high in potassium. (Ignatavicius: Medical-Surgical Nursing, 7th Edition, Chapter 13)

Answer: C. Because hyperkalemia can lead to life-threatening bradycardia, the initial action should be to place the client on a cardiac monitor.

The nurse manager of the medical-surgical unit assigns which client to the LPN/LVN? A. 44-year-old admitted with dehydration who has a heart rate of 126 B. 54-year-old just admitted with hyperkalemia who takes a potassium-sparing diuretic at home C. 64-year-old admitted yesterday with heart failure who still has dependent pedal edema D. 74-year-old who has just been admitted with severe nausea, vomiting, and diarrhea (Ignatavicius: Medical-Surgical Nursing, 7th Edition, Chapter 13)

Answer: C. Because the client with heart failure is the most stable of the four clients, this client is most appropriate to assign to the LPN/LVN.

When reviewing the medical record for a client with polycystic kidney disease who is scheduled for computed tomography (CT) angiography with contrast, it is essential for the nurse to perform which intervention? CHART EXHIBIT History: polycystic kidney disease, diabetes, hysterectomy. Assessment: abdomen distended, negative edema. Medications: Glyburide, Metformin, Synthroid. Diagnostics: BUN 26, Creatinine 1.0, HbA1c 6.9, Glucose 132. A. Obtain a thyroid-stimulating hormone (TSH) level. B. Report the blood urea nitrogen (BUN) and creatinine. C. Hold the metformin 24 hours before and on the day of the procedure. D. Notify provider regarding blood glucose and hemoglobin A1c values. (Ignatavicius: Medical-Surgical Nursing, 7th Edition, Chapter 68)

Answer: C. Before studies with contrast media are performed, the nurse must withhold metformin, which may cause lactic acidosis.

A client with cancer who is taking pain medication states, "I am still having pain." During the assessment, the client does not exhibit any physical manifestations of pain. What does the nurse do next? A. Decreases the client's standard pain medication dose B. Gives the client a placebo and monitors the outcome C. Gives the pain medication as requested D. Withholds the pain medication (Ignatavicius: Medical-Surgical Nursing, 7th Edition, Chapter 5)

Answer: C. Both types of chronic pain (chronic cancer pain and chronic non-cancer pain) do not cause sympathetic reactions. Therefore, some clients do not appear to be in pain, even when they are. Clients with cancer tend to know what medication works for them. The nurse needs to follow the protocol for the client regardless of the client's responses when it is chronic cancer pain.

Two nurses are discussing the important attributes of care coordination. They both know that care coordination should be A. Community based. B. Hospital based. C. Team based. D. Health insurance based. (Giddens: Concepts for Nursing Practice, Concept 47)

Answer: C. Care coordination should be team based, or interdisciplinary. This allows the care to extend over several disciplines for a broader approach. Care that is community based, hospital based, or health insurance based would be too broad and might not serve the patient well.

The nurse manager for an oncology unit is evaluating a newly hired staff nurse. Which action by the nurse is of greatest concern to the nurse manager? A. Asking a client with chest pain if the pain is sharp and stabbing B. Instructing a confused postoperative client about how to use patient-controlled analgesia (PCA) C. Preparing to administer a placebo to a client with chronic back pain D. Requesting that a client with chronic pain describe the specific location of the pain (Ignatavicius: Medical-Surgical Nursing, 7th Edition, Chapter 5)

Answer: C. Current national guidelines from regulatory agencies and nursing organizations indicate that placebos should never be used for clients who are experiencing pain.

Which example demonstrates the nurse performing the skill of evaluation? A. The nurse explains the side effects of the new blood pressure medication ordered for the patient. B. The nurse asks the patient to rate pain on a scale of 0 to 10 before administering the pain medication. C. After completing the teaching, the nurse observes the patient draw up and administer an insulin injection. D. The nurse changes the patient's leg ulcer dressing using aseptic technique. (Potter: Fundamentals of Nursing, 8th Edition, Chapter 21)

Answer: C. Evaluation is one of the most important aspects of clinical care coordination; it involves the determination of patient outcomes. Observing a patient demonstrate teaching is evaluation to ensure that he or she has understood teaching. Answer 2 is not evaluation since it occurs before administering a pain medication. The other options are interventions.

Which of the following would alarm the nurse immediately after return of the client from the operating room for cystoscopy performed under conscious sedation? A. Pink-tinged urine B. Urinary frequency C. Temperature of 100.8 D. Client lethargic (Ignatavicius: Medical-Surgical Nursing, 7th Edition, Chapter 68)

Answer: C. Fever, chills, or an elevated white blood cell (WBC) count suggests infection after an invasive procedure; notify the provider.

A nurse is preparing a client for home care pain management following discharge. Which intervention does the nurse implement? A. Discusses pain-relieving strategies on the day of discharge B. Discusses home care only with the client's family, not with the client C. Offers flexibility in home management of the client's current regimen D. Offers information about end-of-life pain control management (Ignatavicius: Medical-Surgical Nursing, 7th Edition, Chapter 5)

Answer: C. Flexibility will be necessary to adapt to the client's needs and allow for uninterrupted control of the client's pain issues.

When a diabetic client returns to the medical unit after IV urography, all of these interventions are prescribed. Which action will the nurse take first? A. Give lisper (Humalog) insulin, 12 units subcutaneously. B. Request a breakfast tray for the client. C. Infuse 0.45% normal saline at 125 mL/hr. D. Administer captopril (Capote). (Ignatavicius: Medical-Surgical Nursing, 7th Edition, Chapter 68)

Answer: C. Fluids are needed because the dye has an osmotic effect, causing dehydration and potential kidney failure.

An outpatient clinic nurse is recovering a client who had a colonoscopy. The client asks for a drink. How does the nurse respond to the client's request? A. "After I hear bowel sounds, you can have a drink." B. "Twenty (20) minutes after the procedure was completed, you may have some liquids." C. "When you are able to pass flatus (gas), you can have a drink." D. "You can have fluids when you get home and are settled." (Ignatavicius: Medical-Surgical Nursing, 7th Edition, Chapter 55)

Answer: C. Fluids are permitted after the client's peristalsis has returned. This is validated by the client's passing flatus.

A nurse is caring for a client who had a fractured ankle repaired. Twenty minutes after receiving 1.5 mg of hydromorphone (Dilaudid) IV push, the client is slow to respond and has constricted pupils and a respiratory rate of 6 breaths/min. What action does the nurse take initially? A. Calls the care provider for a change in the medication order B. Changes the order to every six (6) hours rather than every four (4) hours C. Gives the client a dose of naloxone (Narcan) 0.4 mg IV D. Performs a cognitive assessment on the client (Ignatavicius: Medical-Surgical Nursing, 7th Edition, Chapter 5)

Answer: C. In an unresponsive client, the nurse should administer Narcan 0.4 mg (diluted in 10 mL) over a 2-minute time period to reverse the action of the opioid analgesic.

Which activity does the RN team leader on a large medical-surgical unit assign to the LPN/LVN? A. Assessment of a client scheduled for surgery who is crying and is expressing fear that the pain will be intolerable B. Assessment of the client using a transcutaneous electrical nerve stimulation unit (TENS) to relieve chronic pain C. Complex dressing changes for a sacral wound for a client with type 2 diabetes who was given prescriptions for pain medication before wound care D. Instructions to a postoperative hip replacement client who has just been placed on patient-controlled analgesia (PCA) for pain relief (Ignatavicius: Medical-Surgical Nursing, 7th Edition, Chapter 5)

Answer: C. LPN/LVN education and scope of practice include working within practice parameters to administer pain medication and to perform dressing changes.

The nurse is taking a health history of a newly admitted patient with a diagnosis Rule/out bowel obstruction. Which of the following is the priority question to ask the patient? A. Describe your bowel movements. B. How often do you have a bowel movement? C. When was the last time you moved your bowels? D. Do you routinely use stool softeners, laxatives, or enemas? (Potter: Fundamentals of Nursing, 8th Edition, Chapter 46)

Answer: C. Lack of a bowel movement is a sign of a bowel obstruction and is a medical emergency.

When caring for the client with uremia, the nurse assesses for which of these symptoms? A. Tenderness at the costovertebral angle (CVA) B. Cyanosis of the skin C. Nausea and vomiting D. Insomnia (Ignatavicius: Medical-Surgical Nursing, 7th Edition, Chapter 68)

Answer: C. Manifestations of uremia include anorexia, nausea, vomiting, weakness, and fatigue.

A nurse is assessing a client with hyponatremia. Which finding requires immediate action? A. Diminished bowel sounds B. Heightened acuity C. Muscular weakness D. Urine output of 35 mL/hr (Ignatavicius: Medical-Surgical Nursing, 7th Edition, Chapter 13)

Answer: C. Muscle weakness in clients with hyponatremia requires immediate action. If muscle weakness is present, immediately check respiratory effectiveness because ventilation is dependent on adequate strength of the respiratory muscles.

A client's urinalysis shows all of the following results. Which result does the nurse report to the health care provider? A. pH 5.8 B. Osmolarity 450 C. Nitrites present D. Sodium 5 mEq/L (Ignatavicius: Medical-Surgical Nursing, 7th Edition, Chapter 68)

Answer: C. Nitrites are not usually present in urine. Many types of bacteria, when present in the urine, convert nitrates (normally found in urine) into nitrites. A positive finding indicates a urinary tract infection. Reference: p. 1482, Physiological Integrity

Which statement is true about assessing pain in the older adult client? A. The nurse should assess for present and past pain. B. Older adults actually believe that expressing pain is acceptable. C. Older adults are at great risk for undertreated pain. D. Older adults usually believe that pain signifies a minor illness. (Ignatavicius: Medical-Surgical Nursing, 7th Edition, Chapter 5)

Answer: C. Older adults are at great risk for undertreated pain because of outdated beliefs by some health care providers about older adults' pain sensitivity, tolerance, and ability to take opioids.

Which substance, produced in the stomach, facilitates the absorption of vitamin B12? A. Glucagon B. Hydrochloric acid C. Intrinsic factor D. Pepsinogen (Ignatavicius: Medical-Surgical Nursing, 7th Edition, Chapter 55)

Answer: C. Parietal cells in the stomach produce intrinsic factor, a substance that facilitates the absorption of vitamin B12. Absence of the intrinsic factor causes pernicious anemia.

You assess four patients. Which patient is at greatest risk for the development of hypocalcemia? A. 56-year-old with acute kidney renal failure B. 40-year-old with appendicitis C. 28-year-old who has acute pancreatitis D. 65-year-old with hypertension and asthma (Potter: Fundamentals of Nursing, 8th Edition, Chapter 41)

Answer: C. People who have acute pancreatitis frequently develop hypocalcemia because calcium binds to undigested fat in their feces and is excreted. This is called steatorrhea. This process decreases absorption of dietary calcium and also increases calcium output by preventing resorption of calcium contained in gastrointestinal fluids.

A client with extensive burn injuries is to be weaned from long-term opioid use. What type of opioid dependence does the nurse expect this client to have? A. Addiction B. Equianalgesia C. Physical dependence D. Pseudoaddiction (Ignatavicius: Medical-Surgical Nursing, 7th Edition, Chapter 5)

Answer: C. Physical dependence occurs in everyone who takes opioids over a period of time. When it is necessary to discontinue opioid analgesia for the client who is opioid dependent, slow tapering (weaning) of the drug dosage lessens or alleviates physical withdrawal symptoms.

The client is in the emergency department (ED) for an inability to void and for bladder distention. What is most important for the nurse to provide to the client? A. Increased oral fluids B. IV fluids C. Privacy D. Health history forms (Ignatavicius: Medical-Surgical Nursing, 7th Edition, Chapter 68)

Answer: C. Provide privacy, assistance, and voiding stimulants, such as warm water over the perineum, as needed, for the client with urinary problems.

A client is admitted with hypokalemia and skeletal muscle weakness. Which assessment does the nurse perform first? A. Blood pressure B. Pulse C. Respirations D. Temperature (Ignatavicius: Medical-Surgical Nursing, 7th Edition, Chapter 13)

Answer: C. Respiratory changes are likely because of weakness of the muscles needed for breathing. Skeletal muscle weakness results in shallow respirations. Thus, respiratory status should be assessed first in any client who might have hypokalemia.

A nurse is explaining the procedure for inserting an indwelling urinary catheter. Which of the following explanations regarding anchoring of the catheter, would be most accurate? A. An indwelling catheter tube is secured to a female patient's abdomen to prevent accidental dislodgment. B. An indwelling catheter tube is secured to the male's inner thigh with a strip of nonallergenic tape or a commercial tube holder. C. It is important to anchor the catheter tubing to minimize the risk for urethral trauma, bladder spasms from traction, and to prevent accidental dislodgment. D. When securing the catheter tubing, slack in the catheter should be avoided to prevent movement and possible tissue injury. (Potter: Evolve Course Module 16)

Answer: C. Securing the catheter will minimize the accidental dislodgment of the catheter. It also minimizes the risk for bleeding, trauma, meatal necrosis, and bladder spasms from pressure and traction. Male patient catheter tubes are attached to the lower abdomen or to top of thigh; female patient tubes are attached to the inner thigh. Allow slack in catheter so movement does not create tension on catheter.

The patient is incontinent, and a condom catheter is placed. The nurse should take which action? A. Secure the condom with adhesive tape B. Change the condom every 48 hours C. Assess the patient for skin irritation D. Use sterile technique for placement (Potter: Fundamentals of Nursing, 8th Edition, Chapter 45)

Answer: C. Skin irritation can occur when the condom is twisted at the drainage tube attachment and obstructs urine drainage.

If a patient has a colostomy in the area known as the "ascending colon," what would the nurse expect of the stool in the colostomy device? A. Stool would be dark. B. Stool would be formed. C. Stool would be loose. D. Stool would have flecks of blood. (Giddens: Concepts for Nursing Practice, Concept 14)

Answer: C. Stool in the ascending colon is watery, or loose. Stool should not be dark or have flecks of blood. This would be an abnormal finding. Stool would not be formed, because the colon has not reabsorbed the water yet.

A 40-year-old male patient has been admitted for abdominal surgery. He has no history of prostate problems. The health care provider has ordered that the patient be catheterized. Which of the following would be an appropriate size catheter for this patient? A. 8 French, 3-mL balloon B. 14 French, 5-mL balloon C. 16 French, 5-mL balloon D. 16 French, 30-mL balloon (Potter: Evolve Course Module 16)

Answer: C. The 16 French, 5-mL balloon is an appropriate catheter for an adult male who has never had prostate surgery. The 8 French, 3-mL balloon is a pediatric-size urinary catheter. The 14 French, 5-mL balloon is an appropriate-size catheter for an adult female. The 16 French, 30 mL balloon is an appropriate-size catheter for an adult male who had prostate surgery.

During change-of-shift report, the day shift staff learns that a client with back surgery has been complaining of increasing lower back pain during the night. It is most appropriate for which day staff member to assess the client's pain? A. LPN/LVN who is responsible for administering medications to the client B. RN nurse manager who is in charge of coordinating care for several units C. RN team leader who is responsible for updating the care plan for the client D. RN who has floated to the unit from the emergency department (Ignatavicius: Medical-Surgical Nursing, 7th Edition, Chapter 5)

Answer: C. The RN team leader should assess this client's level of pain and the need for a change in the plan of care.

A cardiac patient talks with the nurse about bowel elimination. The nurse stresses to the patient not to strain during bowel movements. Straining can put pressure on the vagas nerve and cause bradycardia. This is known as A. First-degree heart block. B. Eupnea. C. Valsalva maneuver. D. Tachypnea. (Giddens: Concepts for Nursing Practice, Concept 14)

Answer: C. The Valsalva maneuver happens when the cardiac patient strains to have a bowel movement. First-degree heart block is not brought on by straining. Eupnea means normal respirations and tachypnea means fast respirations; neither has any connection to straining during a bowel movement.

A nurse is caring for a client who is receiving intravenous (IV) magnesium sulfate (MgSO4). Which assessment parameter is critical? A. 24-hour urine output B. Asking the client about feeling depressed C. Hourly deep tendon reflexes (DTRs) D. Monitoring of serum calcium levels (Ignatavicius: Medical-Surgical Nursing, 7th Edition, Chapter 13)

Answer: C. The client who is receiving IV MgSO4 should be assessed for signs of toxicity every hour by assessment of DTRs.

After receiving change-of-shift report, which client does the RN assess first? A. 26-year-old with nausea and vomiting who complains of dizziness when standing B. 36-year-old with a nasogastric (NG) tube who has dry oral mucosa and is complaining of thirst C. 46-year-old receiving IV diuretics whose blood pressure is 95/52 mm Hg D. 56-year-old with normal saline infusing at 150 mL/hr whose hourly urine output has been averaging 75 mL (Ignatavicius: Medical-Surgical Nursing, 7th Edition, Chapter 13)

Answer: C. The client's history of receiving IV diuretics and having low blood pressure indicates that the client may be experiencing hypoperfusion caused by hypovolemia, and that immediate assessment and interventions are needed.

The client is a 69-year-old woman with uncontrolled diabetes, polyuria, and a blood pressure of 86/46. Which staff member is assigned to care for her? A. LPN/LVN who has floated from the hospital's long-term care unit B. LPN/LVN who frequently administers medications to multiple clients C. RN who has floated from the intensive care unit D. RN who usually works as a diabetic educator (Ignatavicius: Medical-Surgical Nursing, 7th Edition, Chapter 13)

Answer: C. The clinical manifestations suggest that the client is experiencing hypovolemia and possible hypovolemic shock. The RN who floated from the intensive care unit will have extensive experience caring for clients with hypovolemia.

A nurse inserting an indwelling Foley catheter in a female patient advances the catheter and obtains clear yellow urine. What is the next action the nurse should take? A. Inflate the balloon with the prefilled syringe of sterile water in the balloon port. B. Pull gently back on the catheter approximately 1 inch or until resistance is met. C. Advance catheter another 1 to 2 inches and inflate balloon. D. Ask patient to bear down as if to void. (Potter: Evolve Course Module 16)

Answer: C. The female urethra is short. Appearance of urine indicates that catheter tip is in bladder. Advancement of catheter ensures that the inflation balloon is in the bladder and not the urethra. The nurse pulls back gently on the catheter after the balloon is inflated. The nurse may ask the patient to bear down as if to void when initially inserting the catheter - this maneuver relaxes the external urethral sphincter.

The nurse notices that a patient has received oxycodone/acetaminophen (Percocet) (5/325) two tablets PO every 3 hours for the past 3 days. What concerns the nurse the most? A. The patient's level of pain B. The potential for addiction C. The amount of daily acetaminophen D. The risk for gastrointestinal bleeding (Potter: Fundamentals of Nursing, 8th Edition, Chapter 43)

Answer: C. The major adverse effect of acetaminophen is hepatotoxicity. The maximum 24-hour dose is 4 g. It is often combined with opioids (e.g., oxycodone [Percocet]) because it reduces the dose of opioid needed to achieve successful pain control.

A newly admitted client who was in an automobile accident has a concussion and is complaining of pain from a fractured femur and broken fingers. Which staff member does the charge nurse on the orthopedic unit assign to care for this client? A. An experienced RN travel nurse who arrived on the unit this morning B. An LPN/LVN who has worked on the orthopedic unit for 6 years C. The neurology unit RN who has floated to the orthopedic unit D. The RN orthopedic case manager who is responsible for discharge planning (Ignatavicius: Medical-Surgical Nursing, 7th Edition, Chapter 5)

Answer: C. The neurology RN will have the skills and experience needed to assess the neurologic and orthopedic status of this client, as well as the client's pain status.

In the role of client advocate, what does the nurse do first for a client who reports pain? A. Administers pain medication B. Assesses the level of pain C. Believes the client's report of pain D. Calls the provider for a medication order (Ignatavicius: Medical-Surgical Nursing, 7th Edition, Chapter 5)

Answer: C. The nurse's primary role in pain management is to advocate for the client by believing reports of pain.

Which of the following defi ning characteristics is consistent with fluid volume deficit? A. A 1-lb (0.5 kg) weight loss, pale yellow urine B. Engorged neck veins when upright, bradycardia C. Dry mucous membranes, thready pulse, tachycardia D. Bounding radial pulse, fl at neck veins when supine (Potter: Fundamentals of Nursing, 8th Edition, Chapter 41)

Answer: C. The nursing diagnosis fluid volume deficit includes extracellular fluid volume (ECV) deficit, hypernatremia, and clinical dehydration. ECV deficit is characterized by dry mucous membranes, thready pulse, and tachycardia, among other indicators. Weight loss of 1 lb (0.5 kg) in 1 week could indicate fat loss instead of fluid loss. ECV deficit causes dark yellow urine rather than pale yellow, which is normal.

The nurse is caring for a patient with a colostomy. Which intervention is most important? A. Cleansing the stoma with hot water B. Inserting a deodorant tablet in the stoma bag C. Selecting a bag with an appropriate-size stoma opening D. Wearing sterile gloves while caring for the stoma (Potter: Fundamentals of Nursing, 8th Edition, Chapter 46)

Answer: C. The opening of the appliance should be no larger than 0.15 to 0.3 cm (1/16 to 1/8 inch) surrounding the stoma to ensure that the skin around the stoma is protected from the enzymes present in the effluent without impinging the stoma.

The nurse directs the NAP to remove a Foley catheter at 1300. The nurse would check if the patient has voided by: A. 1400. B. 1600 C. 1700. D. 2300. (Potter: Fundamentals of Nursing, 8th Edition, Chapter 45)

Answer: C. The patient may experience urinary retention after removal of the catheter. If 4 hours after Foley removal have elapsed without voiding, it may be necessary to reinsert the Foley.

If a patient's indwelling catheter is removed at 0900, the patient should be due to void by: A. 1900 - 2100 (7:00 PM to 9:00 PM) B. 1100 - 1200 (11:00 AM to 12:00 PM) C. 1500 - 1700 (3:00 PM to 5:00 PM) D. 0930 (9:30 AM) (Potter: Evolve Course Module 16)

Answer: C. The patient should be due to void in 6-8 hours, or by 3:00 to 5:00 PM. If the patient fails to void, nursing measures should be taken (i.e., assess for bladder fullness, provide privacy, assist to normal voiding position, run water). If unsuccessful, the health care provider should be notified.

You are the charge nurse on a surgical unit. You are doing staff assignments for the 3-to-11 shift. Which patient do you assign to the licensed practical nurse (LPN)? A. The patient who transferred out of intensive care an hour ago B. The patient who requires teaching on new medications before discharge C. The patient who had a vaginal hysterectomy 2 days ago and is being discharged tomorrow D. The patient who is experiencing some bleeding problems following surgery earlier today (Potter: Fundamentals of Nursing, 8th Edition, Chapter 21)

Answer: C. The patient with the vaginal hysterectomy is stable and requires care that is within the scope of the LPN. The other three patients need a higher level of care requiring assessment, support, and teaching that are the responsibilities of the registered nurse.

When performing bladder scanning to detect residual urine in a female client, the nurse must first assess which of these? A. Abdominal girth B. Presence of urinary infection C. History of hysterectomy D. Hematuria (Ignatavicius: Medical-Surgical Nursing, 7th Edition, Chapter 68)

Answer: C. The scanner must be in the scan mode for male clients to ensure the scanner subtracts the volume of the uterus from the measurement.

The NAP is applying a condom catheter to the patient. The patient asks, "What is the purpose of the skin preparation solution?" The NAP correctly responds: A. "It is used before condom sheath application as an adhesive to hold the condom catheter on." B. "It is an antiseptic to clean pathogens from the area before applying the condom catheter." C. "The skin preparation solution prevents skin irritation and should be dry before the condom catheter is applied." D. "The skin preparation solution helps the condom catheter to go on more easily, reducing friction, and should still be wet when the sheath is applied." (Potter: Evolve Course Module 16)

Answer: C. The skin preparation solution prevents skin irritation and should be dry before the condom catheter is applied.

Which of the following indicates a reason for notifying the health care provider to get an order for removal of an indwelling catheter? A. The patient states, "My bladder feels so full, it is starting to hurt!" B. The catheter has been in place for 3 days. C. The patient's urine appears cloudy with a foul odor. D. The patient is drinking less than 1500 mL of fluids daily. (Potter: Evolve Course Module 16)

Answer: C. These are symptoms of a UTI. A UTI may be cause for an indwelling catheter to be removed. The health care provider should be notified as a sterile urine specimen may be ordered prior to removing the catheter. An indwelling catheter should be removed as soon as possible after insertion because of the risk for catheter-associated urinary tract infection (CAUTI). If the patient states that his bladder feels very full and is starting to hurt it may indicate that the tubing is kinked or the patient may be lying on the tubing preventing drainage. How long a catheter remains in place is determined by several factors, such as the type of material the catheter is made of, facility policy, reason for the catheter, and whether the patient is experiencing any complications. Often catheters intended for long-term use are changed once a month. Patients who are not drinking sufficient amounts of fluid should be encouraged to drink more. Remember to count IV solution in the fluid intake calculation.

When assessing a 55-year-old patient who is in the clinic for a routine physical, the nurse instructs the patient about the need to obtain a stool specimen for guaiac fecal occult blood testing (gFOBT): A. If patient reports rectal bleeding. B. When there is a family history of polyps. C. As part of a routine examination for colon cancer. D. If a palpable mass is detected on digital examination. (Potter: Fundamentals of Nursing, 8th Edition, Chapter 46)

Answer: C. This is used as a diagnostic screening tool for colon cancer as recommended by the American Cancer Society.

During the nursing assessment a patient reveals that he has diarrhea and cramping every time he has ice cream. He attributes this to the cold nature of the food. However, the nurse begins to suspect that these symptoms are associated with: A. Food allergy. B. Irritable bowel. C. Lactose intolerance. D. Increased peristalsis. (Potter: Fundamentals of Nursing, 8th Edition, Chapter 46)

Answer: C. This patient possibly lacks the enzyme needed to digest milk sugar lactase and therefore is potentially lactose intolerant.

The nurse is teaching the client how to provide a "clean catch" urine specimen. Which statement by the client indicates that teaching was effective? A. "I must clean with the wipes and then urinate directly into the cup." B. "I will have to drink 2 liters of fluid before providing the sample." C. "I'll start to urinate in the toilet, stop, and then urinate into the cup." D. "It is best to provide the sample while I am bathing." (Ignatavicius: Medical-Surgical Nursing, 7th Edition, Chapter 68)

Answer: C. To provide a clean catch urine sample, the client should initiate voiding, then stop, then resume voiding into the container. A midstream collection further removes secretions and bacteria because urine flushes the distal portion of the internal urethra.

Situation: A 70-year-old female is admitted to the hospital with heart failure, shortness-of-breath (SOB), and 3+ pitting edema in her lower extremities. Her current medications are furosemide (Lasix), digoxin (Lanoxin), and an angiotensin-converting enzyme (ACE) inhibitor (Lotensin). She states that she stopped taking her Lasix because she did not think that it was helping her heart failure. Her physician orders furosemide (Lasix) 5 mg IV push. Which client assessment determines that the medication is working? A. Decreased blood pressure (BP) B. Increased heart rate C. Increased urine output D. Weight gain (Ignatavicius: Medical-Surgical Nursing, 7th Edition, Chapter 13)

Answer: C. When giving Lasix, the nurse monitors the client for response to drug therapy, especially weight loss and increased urine output.

The family of a client with chronic hyponatremia asks if the water restriction is a punishment for his uncooperative behavior. What is the nurse's best response? A. "No, limiting fluid intake decreases the risk for kidney failure." B. "No, limiting water intake prevents him from losing too much fluid by vomiting." C. "No, limiting fluid intake keeps his blood from becoming more dilute and causing other complications." D. "No, limiting fluid decreases his sense of thirst and prevents him from drinking liquids that contain an excess of sodium." (Ignatavicius: Medical-Surgical Nursing, 7th Edition, Chapter 13)

Answer: C. When hyponatremia is caused by fluid overload, as well as possible sodium loss, the extra fluid can dilute serum electrolyte concentrations, especially sodium, to dangerously low levels. Appropriate therapy aims to reduce the fluid overload by limiting fluids, increasing urine output, and increasing sodium intake. Reference: p. 182, Psychosocial Integrity

The nurse assesses that the patient has a full bladder, and the patient states that he or she is having difficulty voiding. The nurse would teach the patient to: A. Use the double-voiding technique. B. Perform Kegel exercises. C. Use Credé's method. D. Keep a voiding diary. (Potter: Fundamentals of Nursing, 8th Edition, Chapter 45)

Answer: C. With this method pressure is put on the suprapubic area with each attempted void. The maneuver promotes bladder emptying by relaxing the urethral sphincter.

A patient is using scopolamine (Transderm-Scōp) to prevent motion sickness. About which common side effect should the nurse teach the patient? a. Diarrhea b. Vomiting c. Insomnia d. Dry mouth (Kee: Pharmacology: A Patient-Centered Nursing Process Approach, 8th Edition, Chapter 47)

Answer: D.

The health care provider orders a hypertonic crystalloid IV solution for a 70-year-old patient with cerebral edema. Which assessment should the nurse report immediately to the provider? a. Improved blood pressure, increased urinary output, and stable vital signs b. Hyperglycemia, decreased urinary output, and increased level of consciousness c. Thirst, dry mouth, cool extremities, and decreased urinary output d. Headache, irritability, confusion, widening pulse pressure, and decreased level of consciousness (Kee: Pharmacology: A Patient-Centered Nursing Process Approach, 8th Edition, Chapter 16)

Answer: D.

Situation: A 68-year-old man is admitted to the hospital with dehydration. He has a history of atrial fibrillation, congestive heart failure (CHF), and hypertension. His current medications are digoxin (Lanoxin), chlorothiazide (Diuril), and oral potassium supplements. He tells the nurse that he has had flu-like symptoms for the past week and has been unable to drink for the past 48 hours. The physician requests laboratory specimens to be drawn and an isotonic IV to be started. Which IV fluid does the nurse administer? A. 0.45% saline B. 5% dextrose in 0.45% saline C. 5% dextrose in Ringer's lactate D. 5% dextrose in water (D5W) (Ignatavicius: Medical-Surgical Nursing, 7th Edition, Chapter 13)

Answer: D. 5% dextrose in water (D5W) is an isotonic solution.

The nurse recognizes which patient needs to use a fracture pan for a bowel movement? A. The patient who is obese B. The patient experiencing confusion C. The patient on bed rest D. A patient recovering from hip surgery (Potter: Fundamentals of Nursing, 8th Edition, Chapter 46)

Answer: D. A fracture pan is used for a patient with back or lower-extremity health issues. Because a fracture pan is shallow in comparison to a regular bedpan, the fracture pan prevents disturbing the patient's body alignment.

A client with mild hypokalemia caused by diuretic use is discharged home. The home health nurse delegates which of these interventions to the home health aide? A. Assessment of muscle tone and strength B. Education about potassium-rich foods C. Instruction on the proper use of drugs D. Measurement of the client's urine output (Ignatavicius: Medical-Surgical Nursing, 7th Edition, Chapter 13)

Answer: D. A home health aide may measure the client's intake and output, which then would be reported to the RN.

The nurse has the following assignment. Which client should be encouraged to consume 2 to 3 liters of fluid each day? A. Client with chronic kidney disease B. Client with heart failure C. Client with complete bowel obstruction D. Client with hyperparathyroidism (Ignatavicius: Medical-Surgical Nursing, 7th Edition, Chapter 68)

Answer: D. A major feature of hyperparathyroidism is hypercalcemia, which predisposes to kidney stones; this client should remain hydrated.

The health care provider's order is 1000 mL 0.9% NaCl with 20 mEq K+ intravenously over 8 hours. Which assessment finding causes you to clarify the order with the health care provider before hanging this fluid? A. Flat neck veins B. Tachycardia C. Hypotension D. Oliguria (Potter: Fundamentals of Nursing, 8th Edition, Chapter 41)

Answer: D. Administration of KCl (increased K+ intake) to a person who has oliguria (decreased K+ output) can cause hyperkalemia.

After a colonoscopy, a client reports severe abdominal pain. A nurse obtains these data: temperature 100.2° F (37.9° C), pulse 122, blood pressure 100/45, respirations 44, and O2 saturation 89%. Which request from the health care provider does the nurse implement first? A. Give cefazolin (Ancef) 500 mg IV. B. Infuse normal saline at 200 mL/hr. C. Give morphine sulfate 2 mg IV. D. Provide oxygen @ 6 L/min per nasal cannula (N/C). (Ignatavicius: Medical-Surgical Nursing, 7th Edition, Chapter 55)

Answer: D. Based on the data given, the client may be experiencing complications of colonoscopy such as bleeding or perforation. The most immediate concern involves respiratory status, so the client should be placed on oxygen first.

A 90-year-old client with hypermagnesemia is seen in the emergency department (ED). The ED nurse prepares the client for admission to which inpatient unit? A. Dialysis/Home Care B. Geriatric/Rehabilitation C. Medical-Surgical D. Telemetry/Cardiac Step-Down (Ignatavicius: Medical-Surgical Nursing, 7th Edition, Chapter 13)

Answer: D. Because hypermagnesemia causes changes in the electrocardiogram that may result in cardiac arrest, the client should be admitted to the Telemetry/Cardiac Step-Down unit.

A client is admitted to the nursing unit with a diagnosis of hypokalemia. Which assessment does the nurse complete first? A. Auscultating bowel sounds B. Checking deep tendon reflexes (DTRs) C. Determining the level of consciousness (LOC) D. Obtaining a pulse oximetry reading (Ignatavicius: Medical-Surgical Nursing, 7th Edition, Chapter 13)

Answer: D. Because hypokalemia may cause respiratory insufficiency and respiratory arrest, the client's respiratory status should be assessed first.

Which newly written physician prescription does the nurse administer first? A. Intravenous (IV) normal saline to a client with a serum sodium of 132 mEq/L B. Oral calcium supplements to a client with severe osteoporosis C. Oral phosphorus supplements to a client with acute hypophosphatemia D. Oral potassium chloride (KCl) to a client whose serum potassium is 3 mEq/L (Ignatavicius: Medical-Surgical Nursing, 7th Edition, Chapter 13)

Answer: D. Because minor changes in serum potassium level can cause life-threatening dysrhythmias, the first priority should be to administer potassium supplements to the client with hypokalemia.

A patient starts to experience pain while receiving an enema. The nurse notes blood in the return fluid and rectal bleeding. What action does the nurse take first? A. Administers pain medication B. Slows down the rate of instillation C. Tells the patient to breathe slowly and relax D. Stops the instillation and obtains vital signs (Potter: Fundamentals of Nursing, 8th Edition, Chapter 46)

Answer: D. Bleeding is an unexpected outcome. You should stop the procedure, obtain vital signs, and call the health care provider since this is a medical emergency.

A nurse is assessing a client who has come to the emergency department with acute abdominal pain. Which assessment finding is of greatest concern? A. Bowel sounds are hypoactive in all quadrants. B. The client reports spasmodic cramping. C. The client says, "I have not had a bowel movement for 3 days." D. Bruising is noted around the client's umbilicus. (Ignatavicius: Medical-Surgical Nursing, 7th Edition, Chapter 55)

Answer: D. Bruising around the umbilicus (Cullen's sign) is a sign of intra-abdominal bleeding.

A student nurse asks why care coordination is now a top priority for health system redesign. The nursing instructor's best response is A. "Patients like to be cared for by more than one service agency." B. "Care coordination increases confusion about who is responsible for the patient." C. "Community services are lacking, and care coordination helps to fill the void." D. "Every patient will need coordinated care services at some time in life." (Giddens: Concepts for Nursing Practice, Concept 47)

Answer: D. Care coordination is more prominent in health care design today because patients will need coordination of services to promote optimal health care outcomes. Community services are not always lacking but may be restricted by financial resources. Care coordination should not increase confusion but should rather lessen it. Patients are usually knowledgeable about the service agencies that are available to care for them.

The type of care management approach that coordinates and links health care services to patients and their families while streamlining costs and maintaining quality is: A. Primary nursing. B. Total patient care. C. Functional nursing. D. Case management. (Potter: Fundamentals of Nursing, 8th Edition, Chapter 21)

Answer: D. Case management is a care management approach that coordinates and links health care services to patients and their families while streamlining costs and maintaining quality. Case management is defined as "a collaborative process of assessment, planning, facilitation, and advocacy for options and services to meet an individual's health needs through communication and available resources to promote quality cost-effective outcomes."

A client with chronic arthritis pain tells the nurse, "I take 2 arthritis strength Tylenol (650 mg) every 8 hours." How does the nurse respond? A. "Aspirin would be a better, more effective choice for your pain relief." B. "More Tylenol is needed to provide effective pain relief for you." C. "That is the appropriate dose of Tylenol for your pain." D. "You will need to have routine liver and renal function laboratory tests." (Ignatavicius: Medical-Surgical Nursing, 7th Edition, Chapter 5)

Answer: D. Clients taking Tylenol, especially high doses of it, should be reminded to have routine liver and renal function laboratory testing done. Hepatotoxicity and nephrotoxicity are adverse effects associated with long-term use.

As the nurse, you need to complete all of the following. Which task do you complete first? A. Administer the oral pain medication to the patient who had surgery 3 days ago B. Make a referral to the home care nurse for a patient who is being discharged in 2 days C. Complete wound care for a patient with a wound drain that has an increased amount of drainage since last shift D. Notify the health care provider of the decreased level of consciousness in the patient who had surgery 2 days ago (Potter: Fundamentals of Nursing, 8th Edition, Chapter 21)

Answer: D. Decreased level of consciousness is a high priority. A high priority is an immediate threat to a patient's survival or safety such as a physiological episode of obstructed airway, loss of consciousness, or a psychological episode of an anxiety attack. Completing wound care would be the next priority, but it is not as critical as a change in consciousness. The other options are intermediate- or low-priority activities because they do not pose an immediate threat.

While receiving a blood transfusion, your patient develops chills, tachycardia, and flushing. What is your priority action? A. Notify a health care provider B. Insert an indwelling catheter C. Alert the blood bank D. Stop the transfusion (Potter: Fundamentals of Nursing, 8th Edition, Chapter 41)

Answer: D. Development of chills, tachycardia, and flushing during a blood transfusion is an indication of an acute hemolytic reaction. You stop the transfusion immediately so no more of the incompatible blood reaches the patient.

A nurse is establishing a plan of care for a hospitalized client with chronic pain caused by fibromyalgia. Which nursing action does the nurse delegate to a nursing assistant? A. Application of a transcutaneous electrical nerve stimulation (TENS) device B. Education about nonpharmacologic interventions for pain control C. Referral to available community resources for pain management D. Use of conversation about the client's family to distract the client (Ignatavicius: Medical-Surgical Nursing, 7th Edition, Chapter 5)

Answer: D. Distraction techniques such as conversation, music, and television may be implemented by unlicensed nursing staff members.

Situation: A 77-year-old woman is brought to the emergency department by her family after she has had diarrhea for 3 days. The family tells the nurse that she has not been eating or drinking well, but that she has been taking her diuretics for congestive heart failure (CHF). Her laboratory results include a potassium level of 7.0 mEq/L. What is the primary goal of drug therapy for this client? A. Decreasing cardiac contractility and slowing the heart rate B. Elevating serum potassium levels to a safe range C. Maintaining proper diuresis and urine output D. Restoring fluid balance by controlling the causes of dehydration (Ignatavicius: Medical-Surgical Nursing, 7th Edition, Chapter 13)

Answer: D. Drug therapy for dehydration is directed at restoring fluid balance and controlling the causes of dehydration.

A nurse is planning care for a 72-year-old resident of a long-term care facility who has a history of dehydration. Which action does the nurse delegate to unlicensed assistive personnel (UAP)? A. Assessing oral mucosa for dryness B. Choosing appropriate oral fluids C. Monitoring skin turgor for tenting D. Offering fluids to drink every hour (Ignatavicius: Medical-Surgical Nursing, 7th Edition, Chapter 13)

Answer: D. Encouraging a client to take oral fluids is within the scope of practice for UAP.

The client had IV urography 8 hours ago. Which nursing intervention is the priority for this client? A. Maintaining bedrest B. Medicating for pain C. Monitoring for hematuria D. Promoting fluid intake (Ignatavicius: Medical-Surgical Nursing, 7th Edition, Chapter 68)

Answer: D. Ensure adequate hydration by urging the client to take oral fluid or by giving IV fluids. Hydration reduces the risk for kidney damage.

Which client is at greatest risk for hypernatremia? A. 17 year-old with a serum blood glucose of 189 mg/dL B. 30-year-old on a low-salt diet C. 42-year-old receiving hypotonic fluids D. 54-year old who is sweating profusely (Ignatavicius: Medical-Surgical Nursing, 7th Edition, Chapter 13)

Answer: D. Excessive sweating is a common cause of hypernatremia.

A nurse is instructing a client who is being discharged with a diagnosis of congestive heart failure (CHF). Which client statement indicates a correct understanding of CHF? A. "I can gain 2 pounds of water a day without risk." B. "I should call my provider if I gain more than 1 pound a week." C. "Weighing myself daily can determine if my caloric intake is adequate is effective." D. "Weighing myself daily can reveal increased fluid retention." (Ignatavicius: Medical-Surgical Nursing, 7th Edition, Chapter 13)

Answer: D. Fluid retention may not be visible. Rapid weight gain is the best indicator of fluid retention and overload. Each pound of weight gained (after the first half-pound) equates to 500 mL of retained water. The client should be weighed at the same time every day (before breakfast) on the same scale.

An RN is caring for a client who is severely dehydrated. Which nursing action can be delegated to unlicensed assistive personnel (UAP)? A. Consulting with a health care provider about a client's lab results B. Infusing 500 mL of normal saline over 60 minutes C. Monitoring IV fluid to maintain the drip rate at 75 mL/hr D. Providing oral care every 1 to 2 hours (Ignatavicius: Medical-Surgical Nursing, 7th Edition, Chapter 13)

Answer: D. Frequent oral care is an important intervention for a client with fluid volume deficit and is appropriate to delegate to UAP.

A nurse instructs an older adult client to increase intake of dietary potassium when the client is prescribed which classification of drugs? A. Alpha antagonists B. Beta blockers C. Corticosteroids D. High-ceiling (loop) diuretics (Ignatavicius: Medical-Surgical Nursing, 7th Edition, Chapter 13)

Answer: D. High-ceiling (loop) diuretics are potassium-depleting drugs. The client should increase intake of dietary potassium to compensate for this depletion.

Which of the following would be inappropriate to delegate to NAP? A. Application of a condom catheter. B. Perineal care. C. Emptying a leg bag and recording on I&O record. D. Foley catheter insertion. (Potter: Evolve Course Module 16)

Answer: D. Insertion of a Foley catheter requires the skill of the nurse and should not be delegated to NAP. The NAP may apply a condom catheter, perform perineal care, or empty a leg bag and record the amount of fluid on the I&O report.

An older woman is admitted to the unit after her emergency abdominal surgery on PCA morphine. She has early Alzheimer's disease and is attempting to pull out her IV line. What is the first method the nurse will use to assess this client's pain level? A. Ask her family or caregiver what they think her pain level is. B. Ask the client what her pain level is. C. Observe her body movements and facial expression. D. Assume she has pain and give her a bolus of PCA morphine. (Ignatavicius: Medical-Surgical Nursing, 7th Edition, Chapter 5)

Answer: D. It is important to be proactive in this situation and assume pain is present. The Checklist of Nonverbal Pain Indicators lists common pain indicators to observe and document: -Facial expression (e.g., grimacing, crying) -Verbalizations or vocalizations (e.g., screaming) -Body movements (e.g., restlessness) Reference: p. 49, Health Promotion and Maintenance

A patient returning to the nursing unit after knee surgery is verbalizing pain at the surgical site. The nurse's first action is to: A. Call the patient's health care provider. B. Administer pain medication as ordered. C. Check the patient's vital signs. D. Assess the characteristics of the pain. (Potter: Fundamentals of Nursing, 8th Edition, Chapter 43)

Answer: D. It is necessary to monitor pain on a regular basis along with other vital signs. It is important for the nurse to understand that pain assessment is not simply a number.

An older male patient states that he is having problems starting and stopping his stream of urine and he feels the urgency to void. The best way to assist this patient is to: A. Help him stand to void. B. Place a condom catheter. C. Have him practice Credé's method. D. Initiate Kegel exercises. (Potter: Fundamentals of Nursing, 8th Edition, Chapter 45)

Answer: D. Kegel exercises strengthen pelvic floor muscles and are effective in urine control in patients with urge incontinence and difficulty starting and stopping urination.

A cleansing enema is ordered for a 55-year-old patient before intestinal surgery. The nurse understands that the maximum amount of fluid given is: A. 150 to 200 mL. B. 200 to 400 mL. C. 400 to 750 mL. D. 750 to 1000 mL. (Potter: Fundamentals of Nursing, 8th Edition, Chapter 46)

Answer: D. More than 1000 mL of fluid causes distention to the point of rupturing the bowel.

Which action does the nurse delegate to unlicensed assistive personnel (UAP) helping to care for a client with weight loss and anorexia? A. Document a nutritional assessment. B. Lightly palpate the client's abdomen. C. Monitor the client after endoscopy. D. Obtain a stool specimen. (Ignatavicius: Medical-Surgical Nursing, 7th Edition, Chapter 55)

Answer: D. Obtaining a stool specimen is a skill included in UAP education and is within the UAP role.

During application of the condom catheter, the adhesive strip falls to the floor. What is the nurse's best action? A. Obtain silk tape because it has some ability to stretch. B. Use paper tape in a spiral fashion because it is nonallergenic. C. Use plastic IV tape because it is waterproof preventing slippage. D. Obtain another adhesive strip from condom catheter kit. (Potter: Evolve Course Module 16)

Answer: D. Only the adhesive strip that comes with the catheter should be used. Other tapes are unable to provide the flexibility needed for spiral wrap and may impair circulation of the penis.

A patient with chronic low back pain who took an opioid around-the-clock (ATC) for the past year decided to abruptly stop the medication for fear of addiction. He is now experiencing shaking chills, abdominal cramps, and joint pain. The nurse recognizes that this patient is experiencing symptoms of: A. Addiction. B. Tolerance. C. Pseudoaddiction. D. Physical dependence. (Potter: Fundamentals of Nursing, 8th Edition, Chapter 43)

Answer: D. Physical dependence is a state of adaptation that is manifested by a drug class specific withdrawal syndrome produced by abrupt cessation, rapid dose reduction, decreasing blood level of the drug, and/or administration of an antagonist.

An RN is caring for a client with end-stage liver disease that has resulted in ascites. Which action does the RN delegate to unlicensed assistive personnel (UAP)? A. Assessing skin integrity and abdominal distention B. Drawing blood from a central venous line for electrolyte studies C. Evaluating laboratory study results for the presence of hypokalemia D. Placing the client in a semi-Fowler's position (Ignatavicius: Medical-Surgical Nursing, 7th Edition, Chapter 13)

Answer: D. Positioning the client in a semi-Fowler's position is included within UAP education and scope of practice, although the RN will need to supervise the UAP in providing care and will evaluate the effect of the semi-Fowler's position on client comfort and breathing.

A patient with a cardiac history is taking the diuretic furosemide (Lasix) and is seen in the emergency department for muscle weakness. Which laboratory value do you assess first? A. Serum albumin B. Serum sodium C. Hematocrit D. Serum potassium (Potter: Fundamentals of Nursing, 8th Edition, Chapter 41)

Answer: D. Potassium-wasting diuretics such as furosemide increase potassium urinary output and can cause hypokalemia unless potassium intake also increases. Hypokalemia causes muscle weakness.

After having received 0.2 mg of naloxone (Narcan) intravenous push (IVP), a patient's respiratory rate and depth are within normal limits. The nurse now plans to implement the following action: A. Discontinue all ordered opioids B. Close the room door to allow the patient to recover C. Administer the remaining naloxone over 4 minutes D. Assess patient's vital signs every 15 minutes for 2 hours (Potter: Fundamentals of Nursing, 8th Edition, Chapter 43)

Answer: D. Reassess patients who receive naloxone every 15 minutes for 2 hours following drug administration because the duration of the opioid may be longer than the duration of the naloxone and respiratory depression may return.

The nursing assistive personnel (NAP) is assisting the nurse to insert a Foley catheter on a male patient. In which position should the NAP place the patient? A. Sim's position B. Dorsal recumbent C. Supine with legs adducted D. Supine with legs slightly abducted. (Potter: Evolve Course Module 16)

Answer: D. Sim's position would be appropriate for a female patient with mobility limitations or for a male who cannot lie flat. The dorsal recumbent position would be appropriate for catheterizing a female patient. Legs adducted means the patient's legs are together. The appropriate position for catheterizing a male patient is supine with legs slightly abducted.

A patient has acute gastroenteritis with watery diarrhea. Which statement by this patient would indicate that the nurse's teaching has been effective? A. "I should drink a lot of tap water today." B. "I need to take more calcium tablets today." C. "I should avoid fruits with potassium in them." D. "I need to drink liquids with some sodium in them." (Giddens: Concepts for Nursing Practice, Concept 7)

Answer: D. Sodium-containing fluids are removed from the body by acute diarrhea and must be replaced to prevent an extracellular fluid volume (ECV) deficit. Drinking tap water will not prevent ECV deficit from diarrhea, because tap water does not contain enough sodium to hold the water in the extracellular compartment. Taking calcium tablets is an incorrect answer because hypocalcemia is characteristic of chronic diarrhea rather than acute diarrhea. Restricting fruits is an incorrect answer because diarrhea increases the potassium output and the potassium intake should be increased to balance it.

When teaching a patient about transcutaneous electrical nerve stimulation (TENS), which information do you include? A. TENS works by causing distraction. B. TENS therapy does not require a health care provider's order. C. TENS requires an electrical source for use. D. TENS electrodes are applied near or directly on the site of pain. (Potter: Fundamentals of Nursing, 8th Edition, Chapter 43)

Answer: D. TENS involves stimulation of the skin with a mild electrical current passed through external electrodes. The therapy requires a health care provider order. The TENS unit consists of a battery-powered transmitter, lead wires, and electrodes. Place the electrodes directly over or near the site of pain.

The registered nurse (RN) checks on a patient who was admitted to the hospital with pneumonia. The patient is coughing profusely and requires nasotracheal suctioning. Orders include an intravenous (IV) infusion of antibiotics. The patient is febrile and asks the RN if he can have a bath because he has been perspiring profusely. Which task is appropriate to delegate to the nursing assistant? A. Assessing vital signs B. Changing IV dressing C. Nasotracheal suctioning D. Administering a bed bath (Potter: Fundamentals of Nursing, 8th Edition, Chapter 21)

Answer: D. The bed bath is a skill and task within the knowledge level and tasks appropriate for a nursing assistant. The other tasks are the responsibility of the RN. Assessment, dressing change, and suctioning require assessment and skill that are within the scope of practice of the RN.

The client is scheduled for a cystoscopy later this morning. The consent form is not signed, and the client has not had any preoperative medication. The nurse notes that the provider visited the client the day before. What action does the nurse take? A. Asks the client to sign the informed consent B. Cancels the procedure C. Asks the client's spouse to sign the form D. Notifies the department and the provider (Ignatavicius: Medical-Surgical Nursing, 7th Edition, Chapter 68)

Answer: D. The client may be asked to sign the consent form in the department; notifying both the provider and the department ensures communication across the continuum of care, with less likelihood of omission of information.

A client is being monitored for daily weights. The night nurse asks the nursing assistant for the morning weight, and the assistant replies, "She was sleeping so well, I didn't want to wake her to get her weight." How does the nurse respond? A. "Fast thinking! She really needs to rest after the night she had." B. "Get the information now, or I'll report you for not doing your job." C. "Never mind—I will do it myself." D. "Weigh her now. We need her weight daily, at the same time." (Ignatavicius: Medical-Surgical Nursing, 7th Edition, Chapter 13)

Answer: D. The nurse should educate the nursing assistant as to why obtaining the client's weight at the same time each day is important.

A 53-year-old patient is being treated for hypertension and a history of thrombophlebitis (blood clots). She comes to the clinic complaining, "I have to get up all night to go to the bathroom, and I think my urine looks orange!" What is the nurse's best response? A. "It sounds like you may have a urinary tract infection." B. "Your high blood pressure is adversely affecting your kidneys." C. "Have you tried to restrict your fluid intake?" D. "What medications are you taking and when?" (Potter: Evolve Course Module 16)

Answer: D. The nurse should first assess the patient's medication history before making any interpretation. The patient may be taking diuretics before going to bed or taking other medications that can change the urine's color.

What is the recommended amount of time to leave the catheter clamped when obtaining a urine specimen from an indwelling catheter? A. 15 minutes B. 1 hour C. 2 hours D. 30 minutes (Potter: Evolve Course Module 16)

Answer: D. The recommended amount of time to leave the catheter clamped when obtaining a urine specimen from an indwelling catheter is 30 minutes. This allows time for urine to accumulate for collection without causing bladder distention.

The client receiving insulin and glucose infusion therapy for hyperkalemia now has a serum potassium level of 4.7 mEq/L. What is the nurse's best first action? A. Assess the client's respiratory status and then notify the Rapid Response Team. B. Stop the infusion and discontinue the IV access. C. Continue the infusion at the prescribed rate. D. Slow the infusion and notify the health care provider. (Ignatavicius: Medical-Surgical Nursing, 7th Edition, Chapter 13)

Answer: D. The serum potassium is now in the normal range (3.5 to 5.0 mEq/L), but it is at the higher end of normal. The infusion must be slowed to prevent hypokalemia, and the health care provider is notified to determine the target range for this client's serum potassium level. There is no need to notify the Rapid Response Team or stop the infusion. Reference: p. 187, Safe and Effective Care Environment

The formal nursing leadership model A. Has the same structure in any health care organization. B. Uses one leadership style throughout the organization. C. Always has a chief nursing officer position to lead the nursing staff. D. Adapts to the size and needs of the health care system. (Giddens: Concepts for Nursing Practice, Concept 37)

Answer: D. The size and complexity of the nursing leadership team depend on the size and needs of the health care agency. Using the same structure in any health care organization does not adjust for the specific needs of the health care agency. Using one leadership style throughout the organization does not adjust for the specific needs of the departments in the health care agency. Always having a chief nursing officer position to lead the nursing staff does not adjust to the specific needs of the health care agency.

The postoperative patient has difficulty voiding after surgery and is feeling "uncomfortable" in the lower abdomen. Which action should the nurse implement first? A. Encourage fluid intake B. Administer pain medication C. Catheterize the patient D. Turn on the bathroom faucet as he tries to void (Potter: Fundamentals of Nursing, 8th Edition, Chapter 45)

Answer: D. The sound of running water helps many patients to void through the power of suggestion.

Which technique does the nurse use to obtain a sterile urine specimen from the client with a Foley catheter? A. Disconnects the Foley catheter from the drainage tube and collects urine directly from the Foley B. Removes the existing catheter and obtains a sample during the process of inserting a new Foley C. Uses a sterile syringe to withdraw urine from the urine collection bag D. Clamps the tubing, attaches a syringe to the specimen, and withdraws at least 5 mL of urine (Ignatavicius: Medical-Surgical Nursing, 7th Edition, Chapter 68)

Answer: D. This is the correct technique for obtaining a sterile urine specimen from the client with a Foley catheter.

When assessing a client for pain, acute or chronic, what question does the nurse ask the client to obtain the most data? A. "Did someone do this to you?" B. "Does it hurt badly?" C. "Is the pain really that bad?" D. "When does it hurt?" (Ignatavicius: Medical-Surgical Nursing, 7th Edition, Chapter 5)

Answer: D. This response helps determine precipitating factors to identify the source of pain. It is an open-ended question that requires a descriptive response.

The nurse manager on a surgical unit is making assignments for the day. Who is assigned to check and program the patient-controlled analgesia (PCA) pumps on the unit? A. A pharmacy technician B. One registered nurse (RN) C. One registered nurse (RN) and a certified nursing assistant (CNA) D. Two registered nurses (RNs) (Ignatavicius: Medical-Surgical Nursing, 7th Edition, Chapter 5)

Answer: D. To prevent drug errors, it is recommended that two nurses program the dosing parameters into the PCA delivery device.

The nurse is assisting the NAP to remove a Foley catheter. The nurse should intervene if which of the following actions is noted? The NAP: A. explains the procedure to the patient, regardless of condition or level of awareness. B. connects an empty syringe to the balloon port and allows it to fill passively. C. makes sure the balloon is completely deflated before removing the Foley catheter. D. cleans the patient's perineal area, hands the patient their call light, and removes gloves. (Potter: Evolve Course Module 16)

Answer: D. To prevent the transmission of microorganisms the NAP should remove the used gloves and perform hand hygiene before handing the patient any personal items or the call light. The patient should receive a thorough explanation of the catheter procedure, regardless of condition or level of awareness. The syringe should be allowed to fill by gravity. To prevent trauma to the patient's urethra, the balloon should be completely deflated before removal.

A nurse is planning care for a client with hypocalcemia. Which nursing action is appropriate to delegate to unlicensed assistive personnel (UAP)? A. Collaborating with the dietitian to provide calcium-rich foods for the client B. Evaluating the client's laboratory results C. Implementing Seizure Precautions for the client D. Transferring the client from the bed to a stretcher using a lift sheet (Ignatavicius: Medical-Surgical Nursing, 7th Edition, Chapter 13)

Answer: D. Transferring clients is a nursing skill that is included in UAP education and scope of practice.

An RN is caring for a client admitted with dehydration who requires a blood transfusion. Which nursing action does the RN delegate to unlicensed assistive personnel (UAP)? A. Inserting a small-gauge needle for intravenous (IV) access B. Evaluating a headache that develops during the transfusion C. Explaining to the client the purpose of the blood transfusion D. Obtaining baseline vital signs before blood administration (Ignatavicius: Medical-Surgical Nursing, 7th Edition, Chapter 13)

Answer: D. UAP education includes assessment of vital signs.

Postoperative surgical patients should be given acetaminophen and _______________ in scheduled doses throughout the postoperative course, unless contraindicated. A. Antihistamine. B. Local anesthetic. C. Opioids. D. Nonsteroidal anti-inflammatory drug (NSAID). (Giddens: Concepts for Nursing Practice, Concept 26)

Answer: D. Unless contraindicated, all surgical patients should routinely be given acetaminophen and an NSAID in scheduled doses throughout the postoperative course. Opioid analgesics are added to the treatment plan to manage moderate-to-severe postoperative pain. A local anesthetic is sometimes administered epidurally or by continuous peripheral nerve block.

A client had surgery two (2) days ago and reports having a moderate amount of pain, stating that it is "a 7 on a 1 to 10 scale" of intensity. What intervention has the highest priority in the client's nursing care plan? A. Encouraging diversional activities B. Incorporating ADLs as soon as possible C. Teaching key points of the relaxation response D. Using preemptive analgesia (Ignatavicius: Medical-Surgical Nursing, 7th Edition, Chapter 5)

Answer: D. Use of preemptive analgesia is a technique designed to decrease pain in the postoperative period, decrease the requirements for a postoperative analgesic, prevent morbidity, and decrease the hospital stay.

To minimize the patient experiencing nocturia, the nurse would teach him or her to: A. Perform perineal hygiene after urinating. B. Set up a toileting schedule. C. Double void. D. Limit fluids before bedtime. (Potter: Fundamentals of Nursing, 8th Edition, Chapter 45)

Answer: D. With nocturia the patient has to get up during the night to urinate. Limiting fluids 2 hours before bedtime minimizes nocturia.

When planning an assessment of the urethra, what does the nurse do first? A. Examines the meatus B. Notes any unusual discharge C. Records the presence of abnormalities D. Dons gloves (Ignatavicius: Medical-Surgical Nursing, 7th Edition, Chapter 68)

Answer: D. Before examination begins, body fluid precautions (gloves) must be donned first.

The nurse is caring for a patient who is unable to get out of bed. During the nurse's routine assessment, the nurse notices that urine seems to be pooling in the space at the end and around the condom catheter. The NAP comes to the nurse complaining that the patient's condom catheter has fallen off for the second time today, requiring changing of bed linens. Which is an appropriate response from the nurse? A. "Just leave the condom catheter off because it doesn't seem to be working. I'll see if the doctor will order a Foley catheter." B. "I will contact the health care provider to see if the patient's intravenous (IV) fluids can be decreased so he won't be voiding so often." C. "You can add some additional tape at the base of the patient's penis to hold it in place better." D. "Please apply a new condom catheter, but this time don't leave any space between the tip of the penis and the end of the condom catheter." E. "Let's change the patient from a leg bag to a bedside drainage bag and see if that helps." F. "Let's check the condom catheter size. Perhaps there is one that will fit better." (Potter: Evolve Course Module 16)

Answer: F. If the condom falls off, the nurse should reassess the current condom size, check the manufacturer's size chart, and reapply as necessary. Perineal hygiene should be provided before application of the condom catheter. The patient's fluids should never be limited to decrease urine output. Avoid adding additional tape or overlapping tape because it may impede circulation. Only the tape that is provided by the manufacturer should be used. Space should be left between the glans penis and the condom catheter to allow free flow of urine and avoid irritation of the glans penis. Foley bags are better for the bedridden patient because they have less risk of impairing venous circulation of the leg than a leg bag.

A patient is receiving fluid replacement. The nurse's health teaching with this patient includes which suggestions? (Select all that apply.) a. Measure weight daily. b. Know that thirst means a mild fluid deficit. c. Monitor fluid intake. d. Avoid the use of calcium supplements. e. Monitor output. (Kee: Pharmacology: A Patient-Centered Nursing Process Approach, 8th Edition, Chapter 16)

Answers: A, B, C, D, E.

Elimination changes that result from inability of the bladder to empty properly may cause which of the following? (Select all that apply.) A. Incontinence B. Frequency C. Urgency D. Urinary retention E. Urinary tract infection (Potter: Fundamentals of Nursing, 8th Edition, Chapter 45)

Answers: A, B, C, D, E.

Which situation can cause a client to experience "insensible water loss"? Select all that apply. A. Diarrhea B. Dry, hot weather C. Fever D. Increased respiratory rate E. Nausea F. Mechanical ventilation (Ignatavicius: Medical-Surgical Nursing, 7th Edition, Chapter 13)

Answers: A, B, C, D, F. Nausea with no accompanying vomiting would not cause insensible water loss.

A patient is taking infliximab (Remicade) and asks the nurse what side effects/adverse reactions to expect from this drug. The nurse lists which side effects? (Select all that apply.) a. Fatigue b. Headache c. Chest pain d. Renal damage e. Severe infections (Kee: Pharmacology: A Patient-Centered Nursing Process Approach, 8th Edition, Chapter 25)

Answers: A, B, C, E.

For the patient who is taking acetaminophen (Tylenol), what should the nurse do? (Select all that apply.) a. Monitor routine liver enzyme tests. b. Encourage the patient to check package labels of OTC drugs to avoid overdosing. c. Teach the diabetic patient taking acetaminophen to check blood glucose more frequently. d. Teach the female patient that oral contraceptives can increase the effect of acetaminophen. e. Teach the patient that caffeine decreases the effects of acetaminophen. (Kee: Pharmacology: A Patient-Centered Nursing Process Approach, 8th Edition, Chapter 26)

Answers: A, B, C.

Identify the reasons why a patient with an indwelling catheter may have less than 30 mL per hour of urine in the collection bag: (Select all that apply.) A. The catheter has slipped out of the bladder. B. The patient is severely dehydrated. C. The patient's kidneys are damaged or injured. D. The patient has a UTI. (Potter: Evolve Course Module 16)

Answers: A, B, C. If the catheter is in the urethra outside the bladder, it cannot drain urine. If the patient has insufficient fluid intake, urinary output will be reduced. A sign of renal failure is urine output of less than 30 mL per hour. A urinary tract infection rarely results in urinary output of less than 30 mL per hour.

A nursing student is watching a nurse catheterize a female patient with an indwelling catheter. Which of the following, if it occurs, indicates a break in sterile technique? (Select all that apply.) A. The nurse inserts the urinary catheter, and when urine does not return, removes the catheter and makes a second attempt to locate the urethra with the same catheter. B. The nurse lubricates the catheter and places it back into the sterile tray when it uncoils and touches the bed. C. After the nurse cleans the labia, the labia become slippery and close as the nurse attempts to obtain a clear view of the urethra. D. The nurse advances the catheter another 2.5 to 5 cm (1 to 2 inches) after urine appears, releases the labia, and holds onto the catheter with the nondominant hand. E. The nurse uses forceps and a new cotton ball when cleansing the area, wiping along the far labial fold, the near labial fold, and directly over the center of the urethral meatus. (Potter: Evolve Course Module 16)

Answers: A, B, C. The nurse should never use the same catheter to attempt an insertion a second time because the catheter is contaminated. The nurse should leave the first catheter in the vagina as a landmark and insert another sterile catheter. If the catheter touches the bed, the nurse should obtain a new sterile catheter because the first one has become contaminated. If closure of the labia occurs during cleansing, the cleansing procedure should be repeated because the area has become contaminated. Once urine appears, the nurse should advance the catheter to ensure bladder placement. The nurse is correct in releasing the labia and holding onto the catheter with the nondominant hand because bladder or sphincter contraction may cause accidental expulsion of the catheter. The dominant hand is used to inflate the balloon of the catheter. The nurse also used the correct technique in cleansing the area.

A patient has hypernatremia. Which components are appropriate to include in the nursing care plan? (Select all that apply.) a. Seizure precautions b. Need to keep appointments for laboratory tests and with health care provider c. Signs and symptoms of hypernatremia (muscle cramps, weakness, fatigue) d. Need for fluid restrictions e. Need to read food labels (Kee: Pharmacology: A Patient-Centered Nursing Process Approach, 8th Edition, Chapter 16)

Answers: A, B, D, E.

The nurse has a sterile urinary catheter and sterile gloves. Choose the remaining equipment the nurse will need to insert a straight urethral catheter: (Select all that apply.) A. Sterile cotton balls B. Antiseptic solution C. Sterile urinary collection bag D. Water-soluble lubricant E. Clean cotton balls F. Sterile forceps G. Sterile water in a syringe (without needle) (Potter: Evolve Course Module 16)

Answers: A, B, D, F. Straight urinary catheterization requires aseptic (sterile) technique. The nurse will need five to six sterile cotton balls soaked in antiseptic solution, such as Betadine, to reduce the number of microorganisms present on perineal area. Sterile forceps are used to pick up the antiseptic-saturated cotton balls. Water-soluble lubricant is used to ease insertion of the catheter. Once the catheter is inserted, the nurse will need a container to catch the urine. If a urine specimen is required, the nurse will need a sterile specimen cup. If catheterizing to empty the bladder, the nurse may use a bedpan to collect the urine. A sterile urinary collection bag is used for an indwelling catheter. Sterile water or normal saline in a syringe is used to inflate the balloon on an indwelling catheter and is unnecessary for a straight catheter.

Which of the following strategies focus on improving nurse-physician collaborative practice? (Select all that apply.) A. Inviting the physician to attend the practice council meeting B. Participating in physician morning rounds C. Placing physician photos and names in unit newsletter D. Contacting physician promptly to discuss patient problems E. Providing a list of physician contact numbers to all staff nurses (Potter: Fundamentals of Nursing, 8th Edition, Chapter 21)

Answers: A, B, D. Inviting the physician to attend the practice council meeting, participating in physician morning rounds, and contacting the physician promptly to discuss patient problems improve nurse-physician collaboration by focusing on strategies that are related to professional practice. Collaboration is a process whereby different perspectives are synthesized to better understand complex problems and an outcome that is a shared solution that could not have been accomplished by a single person or organization. Placing physician photos and names in a newsletter and providing physician contact numbers to all staff nurses provide information to the nursing staff and help them identify and contact providers but are not focused specifically on nurses working with physicians to provide patient care.

The nurse is caring for a Hindu patient. Which of the following would be important nursing measures when inserting a urinary catheter? The nurse: (Select all that apply.) A. provides privacy during catheter insertion. B. is of the same gender as the patient. C. uses the right hand to handle urinary secretions. D. avoids putting soiled linens on the bedside table. E. provides all perineal care for the patient while hospitalized. (Potter: Evolve Course Module 16)

Answers: A, B, D. This can be an embarrassing procedure for many patients therefore privacy should be provided through adequate draping and use of bedside screens. Gender-congruent care for cultures emphasizing separate gender roles and female modesty is important such as African, Hispanic, Asian, Islamic, Arabic, Hindu, Jewish Orthodox, and Amish cultures. The nurse should use the left hand to handle urinary secretions. Hindus and Muslims consider the left hand to be unclean. The nurse should avoid placing soiled bed linens on top of the bedside table or surface used for praying or eating. The nurse should provide the patient with the equipment and supplies for cleansing after elimination.

The nurse is reviewing the medication list of a patient with hypokalemia. Which products may be contributing to the cause of this imbalance? (Select all that apply.) a. Cortisone b. Licorice c. Azithromycin d. Estrogen e. Digoxin (Kee: Pharmacology: A Patient-Centered Nursing Process Approach, 8th Edition, Chapter 16)

Answers: A, B, E.

The client is scheduled for intravenous urography. During the assessment, the nurse notes a previous reaction of urticaria, itching, and sneezing to contrast dye. Which precautions does the nurse take? Select all that apply. A. Ensures that an antihistamine and a steroid are prescribed B. Documents the reaction on the chart C. Uses no contrast dye for the procedure D. Cancels the procedure E. Ensures that the health care provider is aware of the reaction (Ignatavicius: Medical-Surgical Nursing, 7th Edition, Chapter 68)

Answers: A, B, E. Contrast will give a more clear picture; if the provider believes it is necessary, suppression of the immune response is an acceptable intervention.

Which client is at increased risk for fluid and electrolyte imbalance? Select all that apply. A. 22-year-old pregnant woman in her third trimester B. 24-year-old male athlete C. 45-year-old man on diuretics D. 47-year-old man traveling to South America in summer E. 76-year-old bedridden woman (Ignatavicius: Medical-Surgical Nursing, 7th Edition, Chapter 13)

Answers: A, B. A pregnant client in the third trimester does have an increase in total body fluids, but this accumulation occurs gradually throughout the pregnancy.

Reasons for lack of urine after inserting a straight catheter include: (Select all that apply.) A. The catheter is outside of the bladder. B. The catheter is inserted in the vagina rather than in the urethra of a female patient. C. The male patient's prostate is preventing urine from exiting the bladder. D. Urethral spasms are preventing urine from exiting the body. (Potter: Evolve Course Module 16)

Answers: A, B. The catheter may be in the urethra ahead of the internal sphincter of the bladder. Catheter malposition may be a cause of lack of urine. Urethral spasms may cause discomfort but will not prevent urine flow with an established catheter. An enlarged prostate may hinder catheter insertion but once inserted, does not prevent urine from exiting the body.

Coordination of health care services is complicated by which of the following? (Select all that apply): A. Involving one or more services. B. Too many community resources. C. Poorly funded social programs. D. Authority for managing services. E. Too many nurses. F. Lack of health care systems. (Giddens: Concepts for Nursing Practice, Concept 47)

Answers: A, C, D, F. Coordination of services is complicated because it involves more than one service, poorly funded social programs, confusing chains of authority for managing services, and a lack of health care systems. Too many community resources and too many nurses are not complicating the health care system.

The nurse is teaching the male patient and family caregiver about the advantages of a condom catheter. Which of the following should be included in the teaching? (Select all that apply.) A. It is relatively safe and noninvasive. B. It ensures complete bladder emptying. C. It is a convenient method of draining urine. D. It is used for male patients who are incontinent. E. It may remain in place for several days to a week, if it remains intact. F. It carries less risk of developing a UTI than an indwelling catheter. (Potter: Evolve Course Module 16)

Answers: A, C, D, F. The external application of a condom catheter is a convenient, safe method of draining urine in male patients. There is less risk of the patient developing a UTI because it is noninvasive. It is suitable for incontinent or comatose patients who still have complete and spontaneous bladder emptying. Unlike a straight or indwelling catheter, the condom catheter cannot ensure complete bladder emptying because it only provides a means for draining the urine that is spontaneously expelled from the body. It should be changed every 24 hours, even if it is intact, but can be replaced.

The nurse is teaching a patient about taking aspirin. Which are important points for the nurse to include? (Select all that apply.) a. Advise the patient to avoid alcohol while taking aspirin. b. Instruct the patient to take aspirin before meals on an empty stomach. c. Instruct the patient to inform the dentist of the aspirin dosage before having dental work. d. Instruct the patient to inform the surgeon of the aspirin dosage before having surgery. e. Suggest that aspirin may be given to children for flu symptoms. (Kee: Pharmacology: A Patient-Centered Nursing Process Approach, 8th Edition, Chapter 25)

Answers: A, C, D.

Which of the following demonstrate that further teaching is required to prevent an infection related to being catheterized? (Select all that apply.) A. An elderly female carries her urinary drainage bag like a purse under her arm as she ambulates. B. A patient drinks an entire pitcher of water over the period of one day. C. As a patient is being transferred in a wheelchair, he places the drainage bag in his lap. D. The NAP places a patient's drainage bag on a lowered side rail or on the floor. E. A female patient keeps her catheter secured to her thigh with tape. (Potter: Evolve Course Module 16)

Answers: A, C, D. The urinary drainage bag should be kept below the level of the bladder to prevent reflux of urine into the bladder. Patients should be instructed to carry the drainage bag below the level of the bladder, and to secure the drainage bag to the side of the wheelchair below the level of the bladder during transfer. The urinary drainage bag should never be placed on a bedside rail because it could accidentally be raised to a height higher than the level of the bladder and urine could reflux into the bladder. The urinary drainage bag should never be placed on the floor; this is to avoid having bacteria enter the system through the drainage port. If allowed, fluids should be encouraged. The catheter should be secured to the patient to prevent trauma to the urethra. Swelling of tissues can impair urine flow and place the patient at further risk for urinary tract infection.

For the patient who is taking nalbuphine (Nubain), what should the nurse do? (Select all that apply.) a. Monitor any changes in respirations. b. Instruct the patient to report bradycardia. c. Administer IV nalbuphine undiluted. d. Explain to the patient to expect an excessive amount of urine output. e. Instruct the patient to avoid alcohol when taking nalbuphine to avoid respiratory depression. (Kee: Pharmacology: A Patient-Centered Nursing Process Approach, 8th Edition, Chapter 26)

Answers: A, C, E.

Which of the following activities can you delegate to nursing assistive personnel (NAP)? (Select all that apply.) A. Measuring oral intake and urine output B. Preparing intravenous (IV) tubing for routine change C. Reporting an IV container that is low in fluid D. Changing an IV fluid container (Potter: Fundamentals of Nursing, 8th Edition, Chapter 41)

Answers: A, C. The registered nurse cannot delegate working with IV tubing or changing an IV infusion to NAP.

A patient with a Foley catheter carries the collection bag at waist level when ambulating. The nurse tells the patient that he or she is at risk for: (Select all that apply.) A. Infection. B. Retention. C. Stagnant urine. D. Reflux of urine. (Potter: Fundamentals of Nursing, 8th Edition, Chapter 45)

Answers: A, D.

Which of the following medications listed in a patient's medication history possibly causes gastrointestinal bleeding? (Select all that apply.) A. Aspirin B. Cathartics C. Antidiarrheal opiate agents D. Nonsteroidal antiinflammatory drugs (NSAIDs) (Potter: Fundamentals of Nursing, 8th Edition, Chapter 46)

Answers: A, D.

The nurse is talking with a patient who was just diagnosed with a urinary tract infection. The patient asks the nurse how to prevent such infections in the future. The nurse identifies some appropriate recommendations for the patient. (Select all that apply): A. Drink 6 to 8 glasses of noncaffeinated fluids daily. B. Exercise daily. C. Increase fiber in the diet. D. Void when the urge is felt. E. Eat fruit twice daily. (Giddens: Concepts for Nursing Practice, Concept 14)

Answers: A, D. Drinking noncaffeinated drinks and voiding when the urge happens are the most appropriate measures for avoiding a urinary tract infection. Increasing fiber, exercising, and eating fruit do nothing to prevent a urinary tract infection.

Assessment findings consistent with intravenous (IV) fluid infiltration include: (Select all that apply.) A. Edema and pain B. Streak formation C. Pain and erythema D. Pallor and coolness E. Numbness and pain (Potter: Fundamentals of Nursing, 8th Edition, Chapter 41)

Answers: A, D. Inadvertent fluid leakage into the interstitial compartment around an IV site can cause swelling, pain from the pressure, pale color, and coolness of the infiltrated area.

The patient is to have an intravenous pyelogram (IVP). Which of the following apply to this procedure? (Select all that apply.) A. Note any allergies. B. Monitor intake and output. C. Provide for perineal hygiene. D. Assess vital signs. E. Encourage fluids after the procedure. (Potter: Fundamentals of Nursing, 8th Edition, Chapter 45)

Answers: A, E.

Which factors place the client at risk for GI problems? Select all that apply. A. Eating a high-fiber diet B. Smoking a half-pack of cigarettes per day C. Socioeconomic status D. Some herbal preparations E. Use of nonsteroidal anti-inflammatory drugs (NSAIDs) (Ignatavicius: Medical-Surgical Nursing, 7th Edition, Chapter 55)

Answers: B, C, D, E. A diet that is high in fiber does not place a client at risk for developing a GI disturbance. High-fiber diets are generally believed to be healthy for most clients.

Situation: A 68-year-old man is admitted to the hospital with dehydration. Initial laboratory results include a potassium level of 2.7 mEq/L. He has a history of atrial fibrillation, congestive heart failure (CHF), and hypertension. His medications are digoxin (Lanoxin), chlorothiazide (Diuril), and potassium supplements. In time, he recovers from his dehydration and low potassium levels. He says to the nurse, "I would like to take fewer medications and eat foods that contain high amounts of potassium." What foods does the nurse recommend? Select all that apply. A. Apples B. Bananas C. Broccoli D. Oranges E. Spinach (Ignatavicius: Medical-Surgical Nursing, 7th Edition, Chapter 13)

Answers: B, C, D, E. Apples are not high, but rather are considered to be low, in potassium.

A 49-year-old woman comes to the emergency department (ED) with reports of black tarry stools that started 2 weeks ago. In taking a GI history, which questions does the nurse ask that pertain to Gordon's Functional Health Patterns? Select all that apply. A. "Are you having any difficulty having sex? How frequently do you have sex?" B. "Do you have any difficulty chewing or swallowing?" C. "Do you have pain, diarrhea, gas, or any other problems? Do any specific foods cause these symptoms for you?" D. "What is your usual bowel elimination pattern? Frequency? Character?" E. "When was your last colonoscopy?" (Ignatavicius: Medical-Surgical Nursing, 7th Edition, Chapter 55)

Answers: B, C, D, E. Sexual difficulties and frequency are not generally affected by GI problems. This would not be a routine question in a GI problem inquiry.

After the 0700 shift report the registered nurse (RN) delegates three tasks to the nursing assistant. At 1300 the RN tells the nursing assistant that he would like to talk to her about the first task that was delegated, which was walking the patient, Mrs. Taylor, earlier that morning. The RN says, "You did a good job walking Mrs. Taylor by 0930. I saw that you recorded her pulse before and after the walk. I saw that Mrs. Taylor walked in the hallway barefoot. For safety, the next time you walk a patient, you need to make sure that the patient wears slippers or shoes. Please walk Mrs. Taylor again by 1500." Which characteristics of good feedback did the RN use when talking to the nursing assistant? (Select all that apply.) A. Feedback is given immediately. B. Feedback focuses on one issue. C. Feedback offers concrete details. D. Feedback identifies ways to improve. E. Feedback focuses on changeable things. F. Feedback is specific about what is done incorrectly only. (Potter: Fundamentals of Nursing, 8th Edition, Chapter 21)

Answers: B, C, D, E. These are characteristics of good feedback. Immediate feedback is not appropriate because the RN did not provide feedback immediately. The nursing assistant performed the task in the morning, but the feedback was not given until the afternoon. Giving feedback about only incorrectly done tasks is not correct because you give both positive and negative feedback (to improve the incorrectly done tasks).

The nurse is collecting a sterile urine specimen for culture and sensitivity from the patient's indwelling Foley catheter. Choose the supplies necessary to carry out the procedure. (Select all that apply.) A. Sterile gloves B. Clean gloves C. Alcohol or disinfectant swab D. 5-mL Luer-Lok syringe E. 20-mL Luer-Lok syringe F. Catheter clamp or elastic band G. Sterile specimen container H. Completed laboratory requisition I. Completed identification label J. Towel/protective barrier K. Bedpan L. Soap, water, washcloth, and towel M. Sterile cotton balls (Potter: Evolve Course Module 16)

Answers: B, C, D, F, G, H, I. The supplies needed to collect a sterile urine specimen for culture and sensitivity include the following: clean gloves; an alcohol swab; a 5-mL Luer-Lok syringe to be attached to needle free port; a catheter clamp or elastic band; a sterile specimen container; a completed laboratory requisition; and an identification label. Although aseptic technique is followed, sterile gloves are unnecessary because a sterile syringe and needle are used to obtain the urine. The gloves are for personal protection from exposure to body fluids. A 20-mL syringe is used for routine urine studies. A towel/protective barrier, bedpan, soap, water, washcloth, towel, and sterile cotton balls are unnecessary for collecting a sterile urine specimen from an indwelling catheter.

Situation: A 77-year-old woman is brought to the emergency department by her family after she has had diarrhea for 3 days. The family tells a nurse that she has not been eating or drinking well, but that she has been taking her diuretics for congestive heart failure (CHF). She is receiving lactated Ringer's solution IV for rehydration. What clinical manifestations does the nurse monitor during rehydration of the client? Select all that apply. A. Blood serum glucose B. Pulse rate and quality C. Urinary output D. Urine specific gravity levels (Ignatavicius: Medical-Surgical Nursing, 7th Edition, Chapter 13)

Answers: B, C, D. Blood glucose changes do not have a direct relation to a client's rehydration status.

The nurse is catheterizing a male patient. Which of the following demonstrates correct understanding of the procedure? (Select all that apply.) A. The patient is placed in a dorsal recumbent position for urinary catheter insertion. B. The patient is placed in a supine position with legs slightly abducted. C. The nurse cleans the urethral meatus using a circular motion from the meatus down to base of glans. D. The nurse applies sterile gloves before opening the antiseptic solution and lubricant. (Potter: Evolve Course Module 16)

Answers: B, C, D. Male patients should be placed in a supine position with legs slightly abducted. The nurse then cleans the urethral meatus using a circular motion from meatus down to the base of the glans. This should be repeated 3 times using a clean cotton ball/stick each time. Everything in the catheter kit is sterile, therefore opening the antiseptic solution and lubricant would not contaminate the sterile gloves.

The nurse in the urology clinic is providing teaching for a female client with cystitis. Which of these should be included in the teaching plan? Select all that apply. A. Cleanse the perineum from back to front after using the bathroom. B. Try to take in 64 ounces of fluid each day. C. Be sure to complete the full course of antibiotics. D. If your urine remains cloudy, call the clinic. E. Expect some flank discomfort until the antibiotic has worked. (Ignatavicius: Medical-Surgical Nursing, 7th Edition, Chapter 68)

Answers: B, C, D. The perineal area should be cleansed from front to back or "clean to dirty" to prevent infection.

The nurse is explaining the case management model to a group of nursing students. Which characteristics best describe the model? (Select all that apply.) A. Case managers provide all patient care. B. Multidisciplinary care plans are used. C. Case managers coordinate discharge planning. D. Staffing is expensive and may not decrease care costs. E. Communication with health care team members is important. F. Model helps to improve patient safety and quality. (Potter: Fundamentals of Nursing, 8th Edition, Chapter 21)

Answers: B, C, E, F. Case management is a care management approach that coordinates and links health care services to patients and their families while streamlining costs and maintaining quality. Activities listed in answers B, C, E, and F are characteristics of the case management model or responsibilities of the case manager. Answers A and D are not characteristics of the case management model. They are typically seen with the primary care model.

The home health nurse should assess a patient who has chronic diarrhea for which fluid and electrolyte imbalances? (Select all that apply). A. Extracellular fluid volume (ECV) excess. B. ECV deficit. C. Hypokalemia. D. Hyperkalemia. E. Hypocalcemia. F. Hypercalcemia. (Giddens: Concepts for Nursing Practice, Concept 7)

Answers: B, C, E. Chronic diarrhea has a high risk of causing ECV deficit, hypokalemia, and hypocalcemia because it increases the fecal output of sodium-containing fluid, potassium, and calcium. Unless the intake of these substances increases appropriately, imbalances will occur. Excesses of ECV, potassium, and calcium are not likely, because the ECV, potassium, and calcium are being removed from the body.

A client reports increasing pain during dressing changes. Which interventions does the nurse recommend for the client? Select all that apply. A. Assistance by the client with the dressing change B. Distraction C. Epidural analgesic D. Music therapy E. Premedication F. Transcutaneous electrical nerve stimulation (TENS) (Ignatavicius: Medical-Surgical Nursing, 7th Edition, Chapter 5)

Answers: B, D, E. Involving the client in an uncomfortable dressing change would tend to increase the client's perception of pain. It is a better tactic to distract the client.

Situation: A 77-year-old woman is brought to the emergency department by her family after she has had diarrhea for 3 days. The family tells a nurse that she has not been eating or drinking well, but that she has been taking her diuretics for congestive heart failure (CHF). Her laboratory results include a potassium level of 7.0 mEq/L. What does the nurse include in the client's medication teaching? Select all that apply. A. Daily weights are a poor indicator of fluid loss or gain. B. Diuretics can lead to fluid and electrolyte imbalances C. Diuretics increase fluid retention. D. Laxatives can lead to fluid imbalance. (Ignatavicius: Medical-Surgical Nursing, 7th Edition, Chapter 13)

Answers: B, D. Daily weight recording is a good indicator of fluid retention. Clients should be taught to weigh themselves at the same time, in the same clothing, and on the same scale.

A patient who is 48 hours post Foley insertion is running a low-grade fever and complains of lower abdominal discomfort, and his urine appears cloudy. The NAP states that his urine had a foul odor when his drainage bag was emptied. Which of the following would be an appropriate nursing action? (Select all that apply.) A. Obtain a urine specimen from the drainage bag. B. Assess the patient for back or flank pain. C. Perform a Hemoccult test on the urine. D. Obtain a health care provider's order and then obtain a sterile urine specimen for culture and sensitivity. (Potter: Evolve Course Module 16)

Answers: B, D. The nurse should assess the patient for systemic symptoms of a UTI. It would also be appropriate to obtain a sterile urine specimen for culture and sensitivity. A urine specimen should only be obtained from the drainage bag immediately after the Foley catheter is inserted. It would be more appropriate to obtain a urine specimen for culture and sensitivity rather than for a Hemoccult test.

Which of the following are true regarding the impact of aging related to urinary elimination? (Select all that apply.) A. The elderly are better able to concentrate urine than the middle-aged adult. B. Aging can affect continence if the patient experiences impaired mobility or decreased muscle tone. C. The elderly are less likely to experience urinary frequency than middle-aged adults because they tend to drink less. D. The elderly are at increased risk for urinary tract infection because of retained urine in the bladder. E. It is part of the normal aging process for elderly patients to become incontinent. (Potter: Evolve Course Module 16)

Answers: B, D. The very young and very old are less able to concentrate urine, placing them at risk for dehydration. The elderly are at an increased risk of urinary incontinence if they have impaired mobility that prevents them from getting to the bathroom in time or from manipulating buttons and zippers. Weak abdominal and pelvic floor muscles impair bladder contraction. Decreased muscle tone increases the risk for urinary incontinence. However, urinary incontinence is not a normal physiological result of the aging process. Urination frequency increases with age with decreased bladder tone. Because the bladder cannot contract as effectively, an older person often retains urine in the bladder after voiding (residual urine). This places the patient at increased risk for bacterial growth and development of UTIs.

Identify the procedures that may be delegated to NAP: (Select all that apply.) A. Insertion of a straight catheter B. Insertion of an indwelling catheter C. Application of a condom catheter D. Collection of a sterile urine specimen from a catheter E. Care of an indwelling catheter

Answers: C, D, E. Trained NAP may apply condom catheters, collect sterile urine specimens, and care for an indwelling catheter.

Nurses discourage patients from straining on defecation primarily because it causes: (Select all that apply.) A. Pain. B. Impaction. C. Hemorrhoids. D. Dysrhythmias. (Potter: Fundamentals of Nursing, 8th Edition, Chapter 46)

Answers: C, D.

Which task is appropriate for a registered nurse (RN) to delegate to the nursing assistant? A. Explaining to the patient the preoperative preparation before the surgery in the morning B. Administering the ordered antibiotic to the patient before surgery C. Obtaining the patient's signature on the surgical informed consent D. Assisting the patient to the bathroom before leaving for the operating room (Potter: Fundamentals of Nursing, 8th Edition, Chapter 21)

Assisting the patient to the bathroom is a skill and task within the knowledge level and tasks appropriate for a nursing assistant. The other tasks are the responsibility of the RN. The RN is responsible for patient teaching, medication administration, and surgical consents.

After a patient has had a Foley catheter for 1 week, a urine specimen may be obtained from the bedside drainage bag. A. True B. False (Potter: Evolve Course Module 16)

False. A urine specimen may be obtained through an indwelling catheter only at the time of catheter insertion, before it is connected to the bedside drainage bag.

Obtaining a sterile urine sample for testing by using a straight catheter can be delegated to NAP. A. True B. False (Potter: Evolve Course Module 16)

False. This procedure is inappropriate to delegate to NAP.

The nurse at a family picnic on a hot day in July is aware that which person is at greatest risk for dehydration while playing softball? A. 32-year-old male cousin who is a professional hockey player B. 28-year-old female cousin who has type 1 diabetes mellitus C. 72-year-old grandmother who is 15 pounds overweight D. 72-year-old grandfather who takes 81 mg of aspirin daily (Ignatavicius: Medical-Surgical Nursing, 7th Edition, Chapter 13)

The thirst mechanism is less sensitive in older adults, making them more at risk for dehydration. Women of any age have less total body water than men of similar sizes and ages, because men have more muscle mass than women and women have more body fat. (Muscle cells contain mostly water and fat cells have little water.) In addition, the grandmother is overweight, with an increased percentage of body fat compared with lean body mass, especially skeletal muscle. An obese person has less total water than a lean person of the same weight because fat cells contain almost no water. The other individuals are not in as great a risk. Reference: p. 171, Physiological Integrity

Obtaining a urine sample from an indwelling catheter requires sterile technique. A. True B. False (Potter: Evolve Course Module 16)

True. Sterile technique is used to prevent contamination of the sterile specimen and to avoid introducing microorganisms into a closed urinary system, which could cause a UTI.


संबंधित स्टडी सेट्स

Chapter 18, Section 4 - Birth of the American Republic

View Set

143 FINAL - Mod 6: Neuro (PRACTICE QUESTIONS)

View Set

PSY 100 LearningCurve: 13a. Classic Perspectives on Personality

View Set

Chapter 7: Networks: Mobile Business

View Set

Lesson 2: Introduction to Construction Technolog Exam

View Set

MACROECONOMICS FINAL- HOMEWORK 1-6

View Set